92
GUIDELINE ANSWERS EXECUTIVE PROGRAMME (New Syllabus) JUNE 2019 MODULE 2 ICSI House, 22, Institutional Area, Lodi Road, New Delhi 110 003 Phones : 41504444, 45341000; Fax : 011-24626727 E-mail : [email protected]; Website : www.icsi.edu

EXECUTIVE PROGRAMME (New Syllabus) · GUIDELINE ANSWERS EXECUTIVE PROGRAMME (New Syllabus) JUNE 2019 MODULE 2 ICSI House, 22, Institutional Area, Lodi Road, New Delhi 110 003 Phones:

  • Upload
    others

  • View
    5

  • Download
    0

Embed Size (px)

Citation preview

Page 1: EXECUTIVE PROGRAMME (New Syllabus) · GUIDELINE ANSWERS EXECUTIVE PROGRAMME (New Syllabus) JUNE 2019 MODULE 2 ICSI House, 22, Institutional Area, Lodi Road, New Delhi 110 003 Phones:

GUIDELINE ANSWERS

EXECUTIVE PROGRAMME(New Syllabus)

JUNE 2019

MODULE 2

ICSI House, 22, Institutional Area, Lodi Road, New Delhi 110 003Phones : 41504444, 45341000; Fax : 011-24626727E-mail : [email protected]; Website : www.icsi.edu

Page 2: EXECUTIVE PROGRAMME (New Syllabus) · GUIDELINE ANSWERS EXECUTIVE PROGRAMME (New Syllabus) JUNE 2019 MODULE 2 ICSI House, 22, Institutional Area, Lodi Road, New Delhi 110 003 Phones:

These answers have been written by competent personsand the Institute hope that the GUIDELINE ANSWERS willassist the students in preparing for the Institute'sexaminations. It is, however, to be noted that the answersare to be treated as model answers and not as exhaustiveand the Institute is not in any way responsible for thecorrectness or otherwise of the answers compiled andpublished herein.

C O N T E N T S Page

MODULE 2

1. Corporate & Management Accounting ... 1

2. Securities Laws & Capital Markets ... 28

3. Economic, Business and Commercial Laws ... 50

3. Financial and Strategic Management ... 68

The Guideline Answers contain the information based on theLaws/Rules applicable at the time of preparation. However,students are expected to be well versed with the amendmentsin the Laws/Rules made upto six months prior to the date ofexamination.

Page 3: EXECUTIVE PROGRAMME (New Syllabus) · GUIDELINE ANSWERS EXECUTIVE PROGRAMME (New Syllabus) JUNE 2019 MODULE 2 ICSI House, 22, Institutional Area, Lodi Road, New Delhi 110 003 Phones:

1 EP–CMA–June 2019

EXECUTIVE PROGRAMME EXAMINATIONJUNE 2019

CORPORATE & MANAGEMENT ACCOUNTINGTime allowed : 3 hours Maximum marks : 100

Total number of questions : 6

NOTE : 1. Answer ALL Questions.2. ALL working notes should be shown distinctly.3. Name of the company is used only for illustration purpose.

PART IQuestion 1

Explain the following :

(a) Parameters those are to be kept in mind during the time of raising and draftingthe Audit queries.

(b) State the advantages of subdivision of Ledger.

(c) Features of International Financial Reporting Standards. (5 marks each)

Answer 1(a)

The following parameters are looked during audit queries:

1. Any questions related to a company which is being audited either by an internalor external auditor.

2. The final touch of the accounts.

3. Audit query is the matter being investigated while examining financial report ofa company.

4. Audit queries are questions asked by an auditor during an investigation. Thesemay be used to gather information to come to a conclusion in the audit.

5. An audit query is an explanation that is required by the audit team on certainpoints that they may have identified during an audit.

6. Audit query is asked by the auditors to justify an action - e.g., invoice accrualprepayment, etc.

7. It is an inquiry from an auditor also known as findings.

8. Audit queries are questions asked by the auditor during an investigation. Theresponse can be used to help the auditor come to a conclusion regarding anaudit.

9. An audit query is when a particular set of data is pulled to audit.

Answer 1(b)

The advantages of subdivision of ledger are:

1. Easy to divide work : As a result of subdivision, records can be maintainedefficiently by the concerned employee.

1

Page 4: EXECUTIVE PROGRAMME (New Syllabus) · GUIDELINE ANSWERS EXECUTIVE PROGRAMME (New Syllabus) JUNE 2019 MODULE 2 ICSI House, 22, Institutional Area, Lodi Road, New Delhi 110 003 Phones:

EP–CMA–June 2019 2

2. Easy to handle : As a result of subdivision, the size and volume of ledger isreduced.

3. Easy to collect information : From the different classes of ledger any particulartype of transaction can be found out easily.

4. Minimizations of mistakes : As a result of subdivision chances of mistakes areminimized.

5. Easy to compute : As a result of subdivision, the calculations in accountingmay be computed quickly which is very helpful to the management.

6. Fixation of responsibility : Due to subdivision, allotment of different types ofwork to different employees is done for which concerned employee will beresponsible.

Answer 1(c)

Features of International Financial Reporting Standards (IFRS) are:

1. Single set of Accounting Standards based on fair value that would enableinternationally to standardize and assure better quality on a global screen.

2. It would also permit international capital to flow more freely, enabling companiesto develop consistent global practices on accounting problems.

3. It would be beneficial to the regulators too, as the complexity associated withthe understanding of various reporting regimes would be reduced.

4. For investors, it gives a better understanding to the financial statements andassess the investment opportunities available in foreign countries other thantheir home country.

5. It also benefits the accounting professionals in a way that they will be able tosell their services in the different parts of world.

Attempt all parts of either Q. No. 2 or Q. No. 2A

Question 2

(a) During the course of the meeting of Board of directors' of Yana Ltd., it has beendecided to forfeit the 500 shares held by Aasha on account of non-payment ofthe call amount of ̀ 4 per share. It has been informed that the face value and thecalled up value per share is ̀ 10. Based on the resolution passed in the meetingof Board of directors, shares of Aasha have been forfeited. In turn, 300 forfeitedshares are reissued at the rate of ` 9 per share. You are required to pass thejournal entries in the books of Yana Ltd. (5 marks)

(b) Aarvi Ltd. has availed a loan of `150 lakh from Bank of India and issued to theBank of India as collateral security - 2,50,000 debentures bearing interest rateof 9%, and of the face value of `100 each.

From the above information, you are required to :

(1) Pass the necessary journal entries in the books of Aarvi Ltd.; and

Page 5: EXECUTIVE PROGRAMME (New Syllabus) · GUIDELINE ANSWERS EXECUTIVE PROGRAMME (New Syllabus) JUNE 2019 MODULE 2 ICSI House, 22, Institutional Area, Lodi Road, New Delhi 110 003 Phones:

3 EP–CMA–June 2019

(2) Show the presentation of the position in the Aarvi Ltd.'s Balance Sheet, asper Schedule III of the Companies Act, 2013. (3 marks)

(c) What are the salient features of Ind AS-103 on "Business Combination" ?(3 marks)

(d) Elaborate the guidelines provided by SEBI with respect to creation of DebentureRedemption Reserve. (5 marks)

(e) The following information is extracted from the Balance Sheet of Chika Ltd. asat 31st March, 2019 :

Name of Account (` Amount in lakh)Term Loans 250Capital Redemption Reserve 210Unpaid Dividends 21Securities Premium Reserve 115Deferred Tax Liabilities 65Interest Received in Advance 10Statement of Profit and Loss (Debit) 118Loans repayable on demand 329% Debentures 400

You are required to make the presentation of above information in the Relevantnotes to the accounts for the year ended as at 31s« March, 2019 as per ScheduleIII of the Companies Act, 2013. The Relevant notes should be related to :(1) Reserves and surplus(2) Non-current liabilities, and(3) Current liabilities. (3 marks)

OR (Alternate question to Q. No. 2)

Question 2A

(i) The following balances were shown in the Balance Sheet of Geeta Ltd. as at31st March, 2019 :

Particulars Amount (` in lakh)500 lakh Equity Shares of `10 each fully paid up 5,00050 lakh, 8% Preference Shares of `10 each `8 paid up 400Capital Reserve 1,950General Reserve 2,500Securities Premium Reserve 4,000Surplus 21012% Debentures 1,800Non-Current Investments at cost 3,200Cash and Bank 2,800

Page 6: EXECUTIVE PROGRAMME (New Syllabus) · GUIDELINE ANSWERS EXECUTIVE PROGRAMME (New Syllabus) JUNE 2019 MODULE 2 ICSI House, 22, Institutional Area, Lodi Road, New Delhi 110 003 Phones:

EP–CMA–June 2019 4

Additional Information :

(1) The company passed a resolution in the Board of directors' meeting, to buy-back 20% of it's equity share capital at `45 per share. For this purpose, itsold it's investments of `2,200 lakh for `2,500 lakh.

(2) Included in it's investments were "Investments in own debentures" costing`150 lakh (face value ̀ 180 lakh). These debentures were cancelled as perBoard of directors' meeting's resolution.

You are required to pass necessary journal entries in the books of Geeta Ltd.(5 marks)

(ii) From the following information, work out the Economic Value Added by VijeLtd.:

Particulars Amount (` in lakh)

Shareholders' Fund 4,200

Long-term Debt 1,800

Trade Payables 750

Net Operating Profit before Interest and Tax 2,150

The average rate of return expected by shareholders' from similar types ofcompanies is 20%. Risk free rate is 10%, and corporate tax rate applicable forVije Ltd. is 30%. (5 marks)

(iii) Beta Ltd. is a subsidiary of Alpha Ltd. The following is the Balance Sheet ofBeta Limited as at 31st March, 2019 :

Particulars Amount (` in lakh)

EQUITY AND LIABILITIES

1. Shareholders' Funds

(a) Share Capital : Equity Shares of `10 each 300

(b) Reserves and Surplus (Statement of Profit and Loss) 250

2. Non-current Liabilities : 8% Debentures 200

3. Current Liabilities 160

Total Equity and Liabilities 910

Assets

1. Non-current Assets :

Tangible Assets :

Land and Building 270

Plant and Machinery 350

2. Current Assets 290

Total Assets 910

Page 7: EXECUTIVE PROGRAMME (New Syllabus) · GUIDELINE ANSWERS EXECUTIVE PROGRAMME (New Syllabus) JUNE 2019 MODULE 2 ICSI House, 22, Institutional Area, Lodi Road, New Delhi 110 003 Phones:

5 EP–CMA–June 2019

On 1st April, 2018, Alpha Ltd. acquired 24 lakh equity shares of Beta Ltd. at acost of `460 lakh. On that date, Statement of Profit and Loss of Beta Ltd.showed a credit balance of `180 lakh and Land & Building was revalued byAlpha Ltd., at 20% above book value of ̀ 300 lakh (but no such adjustments areshown in the books of Beta Ltd.). You are required to calculate :

(1) Cost of Control, and

(2) Minority Interest. (5 marks)

Answer 2(a)Journals of Yana Ltd.

S. No. Particulars Rs. Rs.

1. Equity share capital a/c Dr. 5,000

To Share forfeiture a/c 3,000

To Calls in arrear a/c 2,000

(being forfeiture of 500 shares for non-paymentof first call)

2. Bank a/c Dr. 2,700

Share forfeiture a/c Dr. 300

To Equity share capital a/c 3,000

(being re-issue of 300 forfeited shares@ Rs. 9 per share)

3. Share forfeiture a/c Dr. 1,500

To Capital Reserve A/c 1,500

(being amount already received in respect offorfeited shares @ Rs. 6 per share transferredto Capital reserve a/c after adjustment ofdiscount on re-issue of forfeited share.)

Answer 2(b)(Rs. in lakh)

S. No. Particulars Rs. Rs.

1. Bank a/c Dr. 150

To Bank Loan a/c or Loan from Bank of India a/c 150

(Being a loan of Rs. 150 lakhs taken from Bank of India)

2. Debenture Suspense a/c Dr. 250

To 9% Debenture a/c 250

(being 9% debentures worth Rs. 250 lakh issued ascollateral security for a bank loan taken from Bankof India)

Page 8: EXECUTIVE PROGRAMME (New Syllabus) · GUIDELINE ANSWERS EXECUTIVE PROGRAMME (New Syllabus) JUNE 2019 MODULE 2 ICSI House, 22, Institutional Area, Lodi Road, New Delhi 110 003 Phones:

EP–CMA–June 2019 6

Balance sheet of Aarvi Ltd. as at…………..

(Rs. in lakh)

Particulars Note Amount (Rs.)

EQUITY AND LIABILITIES

Non- current liabilities

Long term borrowings:

9% Debentures of Rs. 100 each 250

Bank loan 150

ASSETS

Non-current assets

Other non-current assets:

Debenture Suspense A/c 250

Current assets

Bank 150

Answer 2(c)

The objective of Ind AS 103 on ‘Business combination’ is to improve the relevance,reliability and comparability of the information that a reporting entity provides in itsfinancial statements about a business combination and its effects. To accomplish that,this Indian Accounting Standard establishes principles and requirements for how theacquirer:

(a) recognizes and measures in its financial statements the identifiable assetsacquired, the liabilities assumed and any non-controlling interest in the acquiree;

(b) recognizes and measures the goodwill acquired in the business combination ora gain from a bargain purchase; and

(c) determines what information to disclose to enable users of the financialstatements to evaluate the nature and financial effects of the businesscombination.

Answer 2(d)

SEBI guidelines on Creation of Debenture Redemption Reserve (DRR)

1. A company has to create DRR in case of the issue of debentures with maturityof more than 18 months.

2. The issuer must create DRR in accordance with the provisions given below.

(a) If debentures are issued for project finance DRR can be created upto thedate of commercial production.

Page 9: EXECUTIVE PROGRAMME (New Syllabus) · GUIDELINE ANSWERS EXECUTIVE PROGRAMME (New Syllabus) JUNE 2019 MODULE 2 ICSI House, 22, Institutional Area, Lodi Road, New Delhi 110 003 Phones:

7 EP–CMA–June 2019

(b) The DRR in respect of debentures issued for project finance may be createdeither in equal installments or higher amounts if profits so permit.

(c) In the case of PCDs, DRR must be created for the non-convertible portionof debenture issues on the same lines as applicable for fully non-convertibledebenture issues.

(d) In respect of convertible issues by new companies, the creation of DRRmust commence from the year the company earns profits for the remaininglife of debentures.

(e) DRR shall be treated as part of general reserve for consideration of bonusissues proposals and for price fixation related to post-tax return.

(f) Company must create DRR equivalent to 50% of the amount of debentureissue before debenture redemption commences. Only after the companyhas actually redeemed 10% of the debenture liability, withdrawal from DRRis permissible only after the company has actually redeemed 10% of thedebenture liability. The requirement of creation of DRR is not applicable tothe issue of debt instruments by infrastructure companies.

Answer 2(e)

Notes to Accounts

1. Reserves and Surplus (Rs. in lakh)

Particulars Amount (Rs.)

Capital Redemption Reserve 210

Securities premium 115

Statement of Profit and Loss (Debit) (118)

Total 207

2. Non Current Liabilities (Rs. in lakh)

Particulars Amount (Rs.)

9% Debentures 400Term Loan 250Deferred tax liability 65

Total 715

3. Current Liabilities (Rs. in lakh)

Particulars Amount (Rs.)

Unpaid Dividends 21

Interest received in advance 10

Loans repayable on demand 32

Total 63

Page 10: EXECUTIVE PROGRAMME (New Syllabus) · GUIDELINE ANSWERS EXECUTIVE PROGRAMME (New Syllabus) JUNE 2019 MODULE 2 ICSI House, 22, Institutional Area, Lodi Road, New Delhi 110 003 Phones:

EP–CMA–June 2019 8

Answer 2A(i)

(Rs. in lakh)

S. No. Particulars Dr. Cr.

1. Bank a/c Dr. 2500

To Investment a/c 2200

To Statement of Profit and Loss 300

(Being investment sold for the purpose of buy-backof equity shares)

2. Equity share capital a/c Dr. 1000

Securities premium a/c (premium payable onbuyback) Dr. 3500

To equity share buy-back a/c 4500

(being the amount due on buy back of equity shares)

3. General reserve a/c Dr. 1000

To Capital redemption reserve a/c 1000

(Being due to buy back of shares, creation of capital

redemption reserve)

4. Equity share buy-back a/c Dr. 4500

To bank a/c 4500

(being payment made for buy back of equity shares)

5. 12% Debenture a/c Dr. 180

To Own debenture a/c 150

To Capital reserve a/c (Profit on cancellation) 30

(being own debentures cancelled at profit)

Alternatively, for entry no. 3 following entry may be passed:

Securities premium a/c Dr. 500

Surplus a/c Dr. 500

To Capital redemption reserve a/c 1000

(Being securities premium and surplus a/c utilisedfor creation of CRR a/c)

Page 11: EXECUTIVE PROGRAMME (New Syllabus) · GUIDELINE ANSWERS EXECUTIVE PROGRAMME (New Syllabus) JUNE 2019 MODULE 2 ICSI House, 22, Institutional Area, Lodi Road, New Delhi 110 003 Phones:

9 EP–CMA–June 2019

Answer 2A(ii)

Capital employed = shareholders’ fund + Long term debt

= Rs. 4200 lakh + Rs. 1800 lakh = Rs. 6000 lakh

Cost of debt after tax = Risk free rate (1-tax rate) = 0.10 (1-0.30) = 7%;

Cost of equity = 20%

Weighted average cost of capital (WACC)

Source of fund Amount (Rs. in lakh) Weight Cost of capital WACC

Equity 4200 0.70 0.20 0.14

Debt 1800 0.30 0.07 0.021

Total 6000 0.161 or 16.10%

Net Operating profit after tax= (Net operating profit before interest and tax –Interest) (1- tax rate)

NOPAT = (Rs. 2150 lakh – Rs. 180 lakh) (1- 0.30) = Rs. 1379 lakh

Calculation of Economic value added

Particulars Rs. in lakh

Net operating profit after tax (NOPAT) 1379

Less : cost of operating capital employed (6000 * 16.10%) (966)

Economic value added (EVA) 413

Answer 2A(iii)

(1) Calculation of Cost of Control

Particulars Rs. in lakh Rs. in lakh

Cost of Investment in shares of Beta Ltd. 460

Less: Paid up share capital held 240

Capital profit- pre-acquisition (80% of Rs. 180 lakh) 144

Profit on revaluation of Land and Building (capital profit) (80% of Rs. 60 lakh) 48 (432)

Cost of control/ Goodwill 28

Page 12: EXECUTIVE PROGRAMME (New Syllabus) · GUIDELINE ANSWERS EXECUTIVE PROGRAMME (New Syllabus) JUNE 2019 MODULE 2 ICSI House, 22, Institutional Area, Lodi Road, New Delhi 110 003 Phones:

EP–CMA–June 2019 10

(2) Computation of Minority Interest

Particulars Rs. in lakh

Paid up value of shares held (Rs. 300 lakh * 20%) 60

Add : share of pre-acquisition profit (Rs. 180 lakh * 20%) 36

Add : share of profit on Revaluation of Land and Building(Rs. 60 lakh * 20%) 12

Add : share of post acquisition profit (Rs. 250 lakh –Rs. 180 lakh – Additional Depreciation Rs. 6 lakh =Rs. 64 lakh * 20%) 12.8

Minority Interest 120.8

Working notes:

Profit on Revaluation and Additional depreciation

Profit on Revaluation of Land and Building = 20% of Rs. 300 lakh = Rs. 60 lakh

Additional Depreciation

Depreciation on Land and Building for 2018-19 = Rs. 300 lakh – Rs. 270 lakh

= Rs. 30 lakh

Rate of Depreciation on land and Building = 30/300 = 10%

Additional Depreciation = Rs. 60 lakh * 10% = Rs. 6 lakh

Question 3

(a) Aman Ltd. issued to the public 16,000, 10% Debentures of `100 each at adiscount of 2%. The amount payable by applicants is as `50 per debenturealong with application and balance amount at the time of allotment. The issuewas fully underwritten by Gama Ltd. for commission at the rate of 1% of nominalvalue of the debentures underwritten. The whole of the issue was subscribed bythe members of the public at large. The company paid underwriting commissionin the form of 10% Debentures of `100 each.

Pass the necessary journal entries in the books of Aman Ltd. for the abovementioned transactions. (5 marks)

(b) Following balances are provided by Nakul Ltd. for the year ended on 31st March,2018 and on 31st March, 2019 :

Amount (` in lakh)

Particulars 31st March, 2018 31st March, 2019

Equity Share Capital 500 560

General Reserves 310 365

Page 13: EXECUTIVE PROGRAMME (New Syllabus) · GUIDELINE ANSWERS EXECUTIVE PROGRAMME (New Syllabus) JUNE 2019 MODULE 2 ICSI House, 22, Institutional Area, Lodi Road, New Delhi 110 003 Phones:

11 EP–CMA–June 2019

Surplus 165 2228% Debentures 350 200Goodwill 60 47Land & Building 430 387Plant & Machinery 300 330Investment (Trading) 48 53Trade Payables 105 122Provision for tax 78 98Outstanding Expenses 18 13Inventories 165 153Trade Receivables 171 206Cash at Bank 52 63Prepaid Expenses 7 5

Additional Information :(1) During the year, an old machine costing ̀ 45 lakh was sold for ̀ 21 lakh. It's

written down value was ̀ 29 lakh.(2) Depreciation charged on plant and machinery @ 15% on the opening balance.(3) There were no purchases or sales of land and building during the year.(4) Provision for tax made during the year was `96 lakh.(5) During the year, premium on redemption of debentures was written-off as

`15 lakh.From the above mentioned information, you are required to prepare aStatement showing the Net Cash Flow from the Operating Activities.

(5 marks)(c) What are the responsibilities of the Trustees of the International Financial

Reporting Standards (IFRS) Foundation ? (5 marks)

Answer 3(a)

Journal of Aman Ltd.

S. No. Particulars Rs. Rs.

1. Bank a/c Dr. 8,00,000

To 10% Debenture Application a/c 8,00,000

(being receipt of application money on 16,000,10% Debentures of Rs. 50 per debenture)

2. 10% Debenture Application a/c Dr. 8,00,000

10% Debenture Allotment a/c Dr. 7,68,000

Discount of issue of Debenture a/c Dr. 32,000

Page 14: EXECUTIVE PROGRAMME (New Syllabus) · GUIDELINE ANSWERS EXECUTIVE PROGRAMME (New Syllabus) JUNE 2019 MODULE 2 ICSI House, 22, Institutional Area, Lodi Road, New Delhi 110 003 Phones:

EP–CMA–June 2019 12

To 10% Debenture a/c 16,00,000

(being allotment of 16,000, 10% debentures ofRs. 100 each at a discount of Rs. 2 perdebenture)

3. Underwriting commission on issue of debenture a/c Dr. 16,000

To Gama Ltd. 16,000

(being underwriting commission @ 1% on Rs. 16lakh, the nominal value of debentures underwritten)

4. Gama Ltd. a/c Dr. 16,000

To 10% Debentures a/c 16,000

(being allotment of 160, 10% debentures of Rs.100 each at par to underwriters by way of paymentof underwriting commission due to them)

5. Bank a/c Dr. 7,68,000

To 10% Debenture allotment a/c 7,68,000

(being receipt of allotment money on 16,000, 10%debentures @ Rs. 48 per debenture)

Answer 3 (b)

Statement showing Net cash outflow from Operating activities for the year ending31st March, 2019

(Rs in lakh)

Particulars Rs.

Surplus as on 31-3-19 222

Less : surplus as on 31-3-18 (165)

Add : Transfer to General Reserves 55

Add : provision for tax 96

Profit before taxes and extraordinary items 208

Adjustment for Depreciation:

Land and Building (Rs. 430 – Rs. 387) 43

Plant and Machinery (15% of Rs 300) 45

Loss on sale of Plant and machinery (Rs. 29 – Rs. 21) 8

Goodwill written off (Rs. 60 – Rs. 47) 13

Premium on redemption of debenture written off 15

Operating profit before working capital changes 332

Page 15: EXECUTIVE PROGRAMME (New Syllabus) · GUIDELINE ANSWERS EXECUTIVE PROGRAMME (New Syllabus) JUNE 2019 MODULE 2 ICSI House, 22, Institutional Area, Lodi Road, New Delhi 110 003 Phones:

13 EP–CMA–June 2019

Working capital changes:

Increase in Investment (Trading) (5)

Decrease in prepaid expenses 2

Decrease in inventory 12

Increase in trade receivables (35)

Increase in trade payables 17

Decrease in outstanding expenses (5)

Cash generated from outstanding operations 318

Income tax paid (76)

Net cash inflow from operating activities 242

Working notes:

Provision for tax a/c

(Rs in lakh)

Particulars Rs. Particulars Rs.

To bank a/c (bal fig) 76 By balance b/d 78

To balance c/d 98 By profit ad loss a/c 96

174 174

Answer 3(c)

The Trustees of the International Financial Reporting Standards (IFRS) Foundationare responsible for the governance and oversight of the International Accounting StandardsBoard (Board).

The Trustees are not involved in any technical matters relating to IFRS Standards.This responsibility rests solely with the Board. The Trustees are accountable to theMonitoring Board, a body of publicly accountable market authorities.

Trustees are appointed for a renewable term of three years. Each Trustee is expectedto have an understanding of, and be sensitive to, international issues relevant to thesuccess of an international organization responsible for the development of high qualityglobal accounting standards for use in the world’s capital markets and by other users.

The Trustees’ responsibilities include the following but not limited to:

• appointing members of the Board, the IFRS Interpretations Committee and theIFRS Advisory Council;

• establishing and amending the operating procedures, consultative arrangementsand due process for the Board, the Interpretations Committee and the AdvisoryCouncil;

Page 16: EXECUTIVE PROGRAMME (New Syllabus) · GUIDELINE ANSWERS EXECUTIVE PROGRAMME (New Syllabus) JUNE 2019 MODULE 2 ICSI House, 22, Institutional Area, Lodi Road, New Delhi 110 003 Phones:

EP–CMA–June 2019 14

• reviewing annually the strategy of the Board and assessing its effectiveness;and

• ensuring the financing of the IFRS Foundation and annually its budget approving.

In exercising their governance responsibilities, the Trustees may reconsider or amendthe Board’s due process or recommend, for example, improvements to the Board’soutreach activities. In addition, the constitution requires the Trustees to undertake aformal, public review of the structure of the IFRS Foundation, its governance arrangementsand its effectiveness in fulfilling the organization’s objectives every five years.

The Trustees are accountable to a Monitoring Board of public authorities. Theirreports to the Monitoring Board are available on the Trustees’ meeting pages.

Question 4

(a) From the following Balance Sheets of Viva Ltd. and Jiyu Ltd., prepare theConsolidated Balance Sheet in the books of Viva Ltd. Viva Ltd. acquired theshares of Jiyu Ltd. as on 1st October, 2018. Balance Sheet as at 31st March,2019 :

Amount (`)Particulars Viva Ltd. Jiyu Ltd.Share Capital 30,00,000 6,00,000General Reserve 4,00,000 —Profit and Loss account 6,00,000 2,10,000Sundry creditors 5,00,000 1,90,000Land and Building 24,00,000 4,00,000Plant and Machinery 4,00,000 4,00,000Current Assets 11,60,000 2,00,000Investments (5,400 shares of

`100 each of Jiyu Ltd.) 5,40,000 —

The profit and loss account of Jiyu Ltd. had a credit balance of `90,000 on 1stApril, 2018. (5 marks)

(b) Prepare a Value Added Statement for the year ended on 31st March, 2019 andalso Reconciliation of Total value added with Profit before Taxes, from theProfit and Loss Statement of Fungi Ltd. :

Particulars Amount (` in '000)Income :Sales 24,400Other Income 508 24,908Expenditure :Operating cost 21,250GST 1,110Interest on Bank Overdraft 75

Page 17: EXECUTIVE PROGRAMME (New Syllabus) · GUIDELINE ANSWERS EXECUTIVE PROGRAMME (New Syllabus) JUNE 2019 MODULE 2 ICSI House, 22, Institutional Area, Lodi Road, New Delhi 110 003 Phones:

15 EP–CMA–June 2019

Interest on 9% Debentures 1,200 23,635Profit before Depreciation 1,273Depreciation 405Profit before Tax 868Provision for tax 320Profit after Tax 548Proposed dividend 48Retained profit 500

The following additional information is provided to you :(1) Sales represent net sales after adjusting discounts and returns.(2) Operating cost includes ̀ 82,50,000 as wages, salaries and other benefits

to the employees.(3) Bank overdraft is for the temporary period only. (7 marks)

Answer 4(a)

Consolidated Balance sheet of Viva Ltd. and its subsidiary Jiyu Ltd.

Particulars Amount (Rs.)

I. EQUITY AND LIABILITIES1. Shareholder’s Fund

(a) Share capital 30,00,000

(b) Reserves and Surplus

Capital reserve 1,35,000

General reserve 4,00,000

Profit and Loss a/c (consolidated) 6,54,000

2. Minority Interest 81,000

3. Non-current liabilities -

4. Current liabilities

Trade payables 6,90,000

TOTAL 49,60,000

II. ASSETS1. Non-current assets

(a) Fixed assets

Land and building 28,00,000

Plant and machinery 8,00,000

2. Current assets 13,60,000

TOTAL 49,60,000

Page 18: EXECUTIVE PROGRAMME (New Syllabus) · GUIDELINE ANSWERS EXECUTIVE PROGRAMME (New Syllabus) JUNE 2019 MODULE 2 ICSI House, 22, Institutional Area, Lodi Road, New Delhi 110 003 Phones:

EP–CMA–June 2019 16

Working notes:

(i) Net worth of Jiyu Ltd.

Rs.

Paid up capital 6,00,000

Opening balance of P&L a/c 90,000

Pre-acquisition profit for 6 months 60,000

Net worth 7,50,000

(ii) Goodwill or Capital Reserve

Cost of investments 5,40,000

Share in net worth of Jiyu Ltd. (90%) 6,75,000

Capital reserve 1,35,000

(iii) Minority Shareholder’s interest

Share in net worth (10%) 75,000

Add : post acquisition profit (60,000 * 10/100) 6,000

Minority shareholder’s interest 81,000

(iv) Consolidated profit

Balance of P&L a/c of Viva Ltd. 6,00,000

Add : share in post acquisition profit (90% of Rs. 60,000) 54,000

6,54,000

Answer 4(b)

Value added Statement of Fungi Ltd.

Rs. in ‘000

Particulars Rs. Rs.

Sales 24,400

Less : Operating cost – cost ofbrought in material and services(Rs. 21,250 – 8,250) 13,000

GST 1,110

Interest on bank overdraft 75 (14,185)

Value added by trading and manufacturing activities 10,215

Add : other income 508

Page 19: EXECUTIVE PROGRAMME (New Syllabus) · GUIDELINE ANSWERS EXECUTIVE PROGRAMME (New Syllabus) JUNE 2019 MODULE 2 ICSI House, 22, Institutional Area, Lodi Road, New Delhi 110 003 Phones:

17 EP–CMA–June 2019

Total value added 10,723

Application of value added %

To pay employees:

Wages, salaries and other benefits 8,250 76.94%

To pay Government : Corporate tax 320 2.98%

To pay provider of capital:

Interest on 9% debentures 1200

Dividends 48 1248 11.64%

To provide for maintenance and expansionof the company:

Depreciation 405

Retained profit 500 905 8.44%

10,723 100%

Reconciliation

Profit before tax 868

Depreciation 405

Wages, salaries and other benefits to employees 8,250

Debenture interest 1,200

10,723

PART II

Question 5

(a) Following data are provided by Mahi Ltd. :

Particulars Product-Kau (`) Product-Vau (`)

Direct material @ `25 per kg. 250 200

Direct labour @ ̀ 48 per hour 240 168

Selling Price 900 667

Variable overheads charged @ ̀ 30 per hour

Raka, Managing Director, would like to decide which product to manufacture inthe following circumstances :

(1) If the monsoon is not up to the mark, in such circumstances the scarcity ofraw material is expected for both products.

Page 20: EXECUTIVE PROGRAMME (New Syllabus) · GUIDELINE ANSWERS EXECUTIVE PROGRAMME (New Syllabus) JUNE 2019 MODULE 2 ICSI House, 22, Institutional Area, Lodi Road, New Delhi 110 003 Phones:

EP–CMA–June 2019 18

(2) On account of e-marketing the sales volume may be a constraint. You arerequired to assist him in decision making, by considering both the situationsin isolation.

(b) What are the disclosure requirements as per Ind AS 102 ?

(c) From the following information, you are required to calculate the Value per EquityShare as on 1st April, 2020 of Udhami Limited :

50,00,000 Equity Shares of `10 each fully paid-up.

20,00,000 Equity Shares of `10 each, `6 paid-up

1,50,000, 9% Debentures of ̀ 100 each.

Expected profit (before interest and tax) for the year ended on 31st March,2020: ̀ 228 lakh.

Expected rate of corporate tax : 31.20%

Normal rate of return on equity shares in the industry, to which the companybelongs to is 15%.

(d) Explain the applicability of Cost Audit as per Rule 4 of the Companies Act,2013. (5 marks each)

Answer 5(a)

Product-Kau (Rs.) Product-Vau (Rs.)

Direct Material @ Rs. 25 per kg 250 200

Direct labour @ Rs. 48 per hour 240 168

Variable Overheads @ Rs. 30 per hour 150 105

Variable cost per unit 640 473

Selling price per unit 900 667

Contribution per unit 260 194

P/V Ratio (Contribution/selling price) 28.89% 29.09%

(1) Direct material is the key factor

Direct material per unit for Kau = 250/25 = 10 kg and for Vau = 200/25 = 8 kg

Contribution per kg for Kau = Rs. 260/10 = Rs. 26 and for Vau = Rs. 194/8 =Rs. 24.25

Hence, Product Kau is recommended.

(2) Sales volume is key factor

Contribution per unit of product Kau is more than that of Product Vau. Hence,Product Kau is recommended.

Page 21: EXECUTIVE PROGRAMME (New Syllabus) · GUIDELINE ANSWERS EXECUTIVE PROGRAMME (New Syllabus) JUNE 2019 MODULE 2 ICSI House, 22, Institutional Area, Lodi Road, New Delhi 110 003 Phones:

19 EP–CMA–June 2019

Answer 5(b)

Ind AS 102 pertains to Accounting for share based payments and requires entitiesto disclose the following:

1. The type and scope of agreements existing during the reporting period.

2. Description of agreements (settlement methods, vesting conditions, etc.).

3. The number and weighted-average exercise price of share options by category(outstanding at the beginning of the reporting period and at the end of the reportingperiod, granted, vested, exercised and forfeited).

4. Average share price of exercised options.

5. Range of exercise prices and remaining contractual life of options outstandingat the end of the reporting period.

6. Valuation method used to estimate the fair value of the awards (model and inputvalues, etc.).

7. The impact on the income statement (i.e., total expense) and the financial position(e.g., carrying amount of liabilities) of share-based payment awards.

Answer 5(c)

Particulars Rs. in lakhs

Expected profit before interest and tax 228

Less : interest (Rs. 1,50,00,000 * 9%) (13.50)

Profit before tax 214.50

Less : Tax @ 31.20% (66.924)

Expected Profit available for equity shareholders 147.576

Expected rate of return = Rs. 147.576 lakh /Rs. 620 lakh = 23.80%

Value per equity shares = Expected rate of return/normal rate of return) * paid up value

Value per equity share (Fully paid up) = (23.80/15) * Rs. 10 = Rs. 15.87

Value per equity share (Rs. 6 paid up) = (23.80/15)* Rs. 6 = Rs. 9.52

Answer 5(d)

The companies required to include cost records in their books of account inaccordance with sub-rule (1) of the Companies (Cost Record and Audit) Rules, 2014,shall be required to get such cost records audited by a cost auditor.

Every company specified in item (A) of rule 3 of the Companies (Cost Record andAudit) Rules, 2014 shall get its cost records audited in accordance with these rules if theoverall annual turnover of the company from all its products and services during theimmediately preceding financial year is rupees fifty crore or more and the aggregate

Page 22: EXECUTIVE PROGRAMME (New Syllabus) · GUIDELINE ANSWERS EXECUTIVE PROGRAMME (New Syllabus) JUNE 2019 MODULE 2 ICSI House, 22, Institutional Area, Lodi Road, New Delhi 110 003 Phones:

EP–CMA–June 2019 20

turnover of the individual product or products or service or services for which cost recordsare required to be maintained under rule 3 of the Companies (Cost Record and Audit)Rules, 2014 is rupees twenty five crore or more.

Every company specified in item (B) of rule 3 of the Companies (Cost Record andAudit) Rules, 2014 shall get its cost records audited in accordance with these rules if theoverall annual turnover of the company from all its products and services during theimmediately preceding financial year is rupees one hundred crores or more and theaggregate turnover of the individual product or products or service or services for whichcost records are required to be maintained under rule 3 is rupees thirty five crores ormore.

The requirement for cost audit under these rules shall not apply to a company whichis covered in rule 3; and-

(i) whose revenue from exports, in foreign exchange, exceeds seventy five percent of its total revenue; or

(ii) which is operating from a special economic zone;

(iii) which is engaged in generation of electricity for captive consumption throughCaptive Generating Plant. For this purpose, the term “Captive Generating Plant”shall have the same meaning as assigned in rule 3 of the Electricity Rules,2005”. Even for regulated sectors like Telecommunication, Electricity, Petroleumand Gas, Drugs and Pharma, Fertilizers and Sugar cost audit requirement hasbeen made subject to a turnover based threshold of Rs. 50 crores for all productand services and Rs. 25 crores for individual product or services. For non-regulated sector the threshold is Rs. 100 crores and Rs. 35 crores, respectively.

Attempt all parts of either Q. No. 6 or Q. No. 6A

Question 6

(a) The following details are provided by Trupti Engineering Ltd. with respect totheir product, named as Prima :

Level of activity (Units) 1,000 2,000

Cost per unit (`)

Direct materials 4.00 4.00

Direct labour 3.00 3.00

Production overheads 3.50 2.50

Selling overheads 1.00 0.50

Total 11.50 10.00

You are required to compute the total fixed cost of Trupti Engineering Ltd. andalso variable cost per unit of the product Prima. (5 marks)

(b) Explain the components of the total risk of a security. Provide necessaryexamples for each component of risks. (5 marks)

Page 23: EXECUTIVE PROGRAMME (New Syllabus) · GUIDELINE ANSWERS EXECUTIVE PROGRAMME (New Syllabus) JUNE 2019 MODULE 2 ICSI House, 22, Institutional Area, Lodi Road, New Delhi 110 003 Phones:

21 EP–CMA–June 2019

(c) What do you understand by Shareholder Value Added (SVA) ? What are thebenefits of adopting SVA ? (5 marks)

(d) You have applied for the position of Assistant Company Secretary in SashvatiLtd. As part of the pre-interview scanning, the following information has beenprovided to you :

Sales `1,825 lakh

Net Worth to Sales 90%

Current Liabilities to Net Worth 20%

Total Debt to Net Worth 50%

Receivables average collection period 44 Days

Inventory Turnover (Based on Cost of Goods Sold) 4.839 Times

Number of days in a year 365

Ratio of inventory, receivables and cash & bank balancein current assets 3:2:1

Cash sales are 1/4th of credit sales

As part of scanning exercise, you are required to calculate and inform to theinterviewer, following values :

(1) Non-current debt (as per the Companies Act, 2013),

(2) Receivables,

(3) Current assets, and

(4) Cost of goods sold. (5 marks)

OR (Alternate question to Q. No. 6)

Question 6A

(i) Roma Ltd. has to choose one of the machines from two mutually exclusivemachines.

Two proposals are : Machine Rani and Machine Prince. The required amount ofcash outlays for Machine Rani is `8,35,000; while for Machine Prince is`9,00,000. The current rate of yield on Government of India's bond is 7%, andthe management is of the opinion to use the same as discount rate. The expectednet cash inflows by deployment of both the machines are as under :

Amount (`)Year end Machine Rani Machine Prince

1 3,60,000 4,05,0002 3,50,000 3,60,0003 2,50,000 3,50,000

Page 24: EXECUTIVE PROGRAMME (New Syllabus) · GUIDELINE ANSWERS EXECUTIVE PROGRAMME (New Syllabus) JUNE 2019 MODULE 2 ICSI House, 22, Institutional Area, Lodi Road, New Delhi 110 003 Phones:

EP–CMA–June 2019 22

You have to advise the management with respect to selection of either MachineRani or Machine Prince. The Present value interest factor for the year 1, 2 and3 are 0.9346, 0.8734 and 0.8163 respectively, for 7% discount rate. (5 marks)

(ii) The following information is provided by Muni Ltd. :

Sales Budget for the year 1st April, 2019 to 31st March, 2020

Quarter Product-Pixel (units) Product-Quita (units)

1st Quarter 25,000 23,000

2nd Quarter 30,000 25,000

3rd Quarter 40,000 32,000

4th Quarter 48,000 40,000

The additional information is provided with respect to inventory management asunder :

(1) Expected stock position as on 1st April, 2019 : Product Pixel and Quita as25% and 80% of expected sales of 1st Quarter of the next financial yearrespectively.

(2) Expected stock position at the end of 1st, 2nd and 3rd Quarter will be 25%of next Quarter's sales for both the products — Pixel and Quita.

(3) Expected stock position as on 31st March, 2020 will be : Product Pixel —28,000 units and Product Quita — 15,000 units.

You are required to prepare Quarter-wise Production Budget for the year 2019-20, for both the products separately. (5 marks)

(iii) What are the essential steps to be taken to have an effective managementreporting system in the firm ? (5 marks)

(iv) Calculate the Weighted Average Cost of Capital of Makkhu Ltd. from the followinginformation :

Particulars Amount in `

4,00,000 Equity Shares of `10 each fully paid-up 40,00,000

80,000, 9% Preference Shares of ̀ 10 each fully paid-up 8,00,000

Retained Earnings 32,00,000

20,000, 8% Debentures of ̀ 100 each 20,00,000

Market price of Equity shares 45

Average Earning per share of last three years 9

Corporate Income tax rate applicable to Makkhu Ltd. 30%

Cost of Equity can be considered as Cost of Retained Earnings. (5 marks)

Page 25: EXECUTIVE PROGRAMME (New Syllabus) · GUIDELINE ANSWERS EXECUTIVE PROGRAMME (New Syllabus) JUNE 2019 MODULE 2 ICSI House, 22, Institutional Area, Lodi Road, New Delhi 110 003 Phones:

23 EP–CMA–June 2019

Answer 6(a)

Variable production overhead per unit = 1000-2000

3.50) x (1000 - 2.50) x (2000 = Rs. 1.50 per unit

Fixed production overheads = (1000*3.50) – (1000*1.50) = Rs. 2000

Fixed selling overheads = 1000*1 = Rs. 1000

Hence, total variable cost per unit = Rs. 4 + Rs. 3 + Rs. 1.50 = Rs. 8.50 per unitand Total Fixed cost = Rs. 2000 + Rs. 1000 = Rs. 3000

Answer 6(b)

Total risk of a security comprises two components:

(i) Systematic Risk or Non-Diversifiable Risk

Systematic risk is the risk which is caused by factors beyond the control ofspecific company, such as general factors in the market, GDP, inflation, interestrates, tax policy, government policy, etc. These factors affect all the companiesand cause variability in their returns. Systematic risk cannot be reduced byholding an efficiently diversified portfolio. Therefore, systematic risk is that partof total risk which cannot be eliminated by diversification. However, the systematicrisk may be managed by ensuring that the portfolio includes a variety assetclasses, like fixed income, cash and real estate, each of which will react differentlyin the event of a major systemic change.

Systematic risk of a security is indicated by beta coefficient (ß). ß captures thesensitivity of a security’s return with respect to market return.

(ii) Unsystematic Risk or Diversifiable Risk

It is that part of total risk which is diversifiable. It is caused by factors which arewithin the control of a specific company, such as management, operationalefficiency, labour conditions and financial leverage. The sources of unsystematicrisk are business risk and financial risk. Securities which are less than perfectlyand positively correlated can be combined together to diversify away unsystematicrisk.

It can be observed that unsystematic risk can be reduced to zero in an efficientlydiversified portfolio; hence the only relevant risk in such a portfolio is systematicrisk.

As per CAPM there is a linear and positive relationship between expected returnand systematic risk measured by ß. CAPM is used to estimate expected returnfrom a security or portfolio. ß measures the sensitivity of a security’s returns tothe return of market portfolio. Some securities are less sensitive while othersare more. Hence ß of different securities and portfolios are also different.Moreover, unsystematic risk of a security can be diversified away, hence investorwill not receive any return or risk premium for bearing unsystematic risk. Theinvestor will receive risk premium only for the systematic risk identified by ß.

Page 26: EXECUTIVE PROGRAMME (New Syllabus) · GUIDELINE ANSWERS EXECUTIVE PROGRAMME (New Syllabus) JUNE 2019 MODULE 2 ICSI House, 22, Institutional Area, Lodi Road, New Delhi 110 003 Phones:

EP–CMA–June 2019 24

Answer 6(c)

Shareholder Value Added (SVA) represents the economic profits generated by abusiness above and beyond the minimum return required by all providers of capital.“Value” is added when the overall net economic cash flow of the business exceeds theeconomic cost of all the capital employed to produce the operating profit.

Therefore, SVA integrates financial statements of the business (profit and loss,balance sheet and cash flow) into one meaningful measure.

The SVA approach is a methodology which recognizes that equity holders as wellas debt financiers need to be compensated for the bearing of investment risk. The SVAmethodology is a highly flexible approach to assist management in the decision-makingprocess. Its applications include performance monitoring, capital budgeting, output pricingand market valuation of the entity.

The benefits of moving towards SVA include:

1. Overall, value-based performance measures will result in greater accountabilityfor the investment of new capital, as well as for the use of existing investments.

2. Organization will have the opportunity to apply a meaningful private sectorbenchmark to evaluate performance.

3. Managers will be provided with an improved focus on maximizing shareholdervalue.

Answer 6(d)

(1) Non-current Debt

Net worth = 90% of sales of Rs. 1825 lakh = Rs. 1642.50 lakh

Current liabilities = 20% of the Net worth of Rs. 1642.50 lakh = Rs. 328.50 lakh

Total debt = 50% of Net worth of Rs. 1642.50 lakh = Rs. 821.25 lakh

Hence, Non current debt = Rs. 821.25 lakh – Rs. 328.50 lakh = Rs. 492.75 lakh

(2) Receivables

Collection period = (Receivables * 365)/ Credit sales

or Receivables = (Collection period * Credit sales)/365= (44* Rs. 1460 lakh)/365

= Rs. 176 lakh

Note : Credit sales = Total sales * 100/125 = Rs. 1460 lakh

(3) Current assets

Current assets = Receivables * 6/2 = Rs. 176 lakh * 6/2 = Rs. 528 lakh

(4) Cost of goods sold

Cost of goods sold = Inventory * inventory turnover

= (Rs. 528 lakh * 3/6) * 4.839 = Rs. 1277.496 lakh

Page 27: EXECUTIVE PROGRAMME (New Syllabus) · GUIDELINE ANSWERS EXECUTIVE PROGRAMME (New Syllabus) JUNE 2019 MODULE 2 ICSI House, 22, Institutional Area, Lodi Road, New Delhi 110 003 Phones:

25 EP–CMA–June 2019

Answer 6A(i)

Appraisal of machines

Machine Rani

Year end Cash flow (Rs.) P.V. Factor @ 7% P.V. (Rs.)

1 3,60,000 0.9346 3,36,456

2 3,50,000 0.8734 3,05,690

3 2,50,000 0.8163 2,04,075

8,46,221

Less : cash outlay (8,35,000)

NPV 11,221

Machine Prince

Year end Cash flow (Rs.) P.V. Factor @ 7% P.V. (Rs.)

1 4,05,000 0.9346 3,78,513

2 3,60,000 0.8734 3,14,424

3 3,50,000 0.8163 2,85,705

9,78,642

Less : cash outlay (9,00,000)

NPV 78,642

Decision : as the NPV of Machine Prince is higher, therefore Machine Prince isPreferable.

Answer 6A(ii)

Production Budget for year 2019-20

Product Pixel(Amount in Rs.)

Particulars 1st quarter 2nd quarter 3rd quarter 4th quarter

Budget sales 25,000 30,000 40,000 48,000

Add : closing stock 7500 10,000 12,000 28,000

32,500 40,000 52,000 76,000

Less : opening stock (6250) (7500) (10,000) (12,000)

Budgeted production 26,250 32,500 42,000 64,000

Page 28: EXECUTIVE PROGRAMME (New Syllabus) · GUIDELINE ANSWERS EXECUTIVE PROGRAMME (New Syllabus) JUNE 2019 MODULE 2 ICSI House, 22, Institutional Area, Lodi Road, New Delhi 110 003 Phones:

EP–CMA–June 2019 26

Product Quita

(Amount in Rs.)

Particulars 1st quarter 2nd quarter 3rd quarter 4th quarter

Budget sales 23,000 25,000 32,000 40,000

Add: closing stock 6250 8,000 10,000 15,000

29,250 33,000 42,000 55,000

Less : opening stock (18,400) (6,250) (8,000) (10,000)

Budgeted production 10,850 26,750 34,000 45,000

Answer 6A(iii)

Steps towards implementing an effective management reporting system in the firmare:

1. Discovery : Early identification and involvement by shareholders, and effectivelycommunicate the vision across the organization and to the appropriatestakeholders. Identify and access the data sources available to meet the specificdata needs.

2. Analysis : Understand the level of effort involved in creating each report. Developdata definitions to ensure everyone interprets it the same way. Keep the datatight and prioritize the information, and understand what the data says. Further,encourage quick decision making abilities of the recipient.

3. Report creation and delivery : Create the reports and determine the appropriatedelivery method of the data. Reports should be concise and comprehensive,but not overwhelming. Pay close attention while formatting and reporting,considering the specific needs of the users.

4. Implementation : Develop each report separately, and establish a governanceprocess and policies to ensure data health across the organization. Define accesscontrol requirements, detailing who should have rights to access information.Minimize work by leveraging implemented software to create repeatable processesand automate report jobs for predetermined times throughout the month.

5. Access point : Create a common place for users to access data, such as aWeb portal or SharePoint site.

6. Feedback : Collect comments and suggestions from users to discover ways tocontinuously improve the data and process.

7. In-house IT capabilities : Ensure that in-house Information Technology (IT)personnel can manage necessary systems and issues if they arise.

Answer 6A(iv)

Cost of equity (Ke) = (EPS/MPS)*100 = (9/45)*100 = 20%

Cost of Preference shares (Kp) = 9%

Page 29: EXECUTIVE PROGRAMME (New Syllabus) · GUIDELINE ANSWERS EXECUTIVE PROGRAMME (New Syllabus) JUNE 2019 MODULE 2 ICSI House, 22, Institutional Area, Lodi Road, New Delhi 110 003 Phones:

27 EP–CMA–June 2019

Cost of retained earnings (Kr) = 20%

Cost of Debt (Kd) = r(1-t) = 8(1-0.30) = 5.6%

Calculation of WACC

Source of Funds Amount (Rs.) Weight Cost of capital WACC

Equity shares 40,00,000 0.4 0.20 0.08

Preference shares 8,00,000 0.08 0.09 0.0072

Retained earnings 32,00,000 0.32 0.20 0.064

Debentures 20,00,000 0.20 0.056 0.0112

Total 1,00,00,000 0.1624 or16.24%

***

Page 30: EXECUTIVE PROGRAMME (New Syllabus) · GUIDELINE ANSWERS EXECUTIVE PROGRAMME (New Syllabus) JUNE 2019 MODULE 2 ICSI House, 22, Institutional Area, Lodi Road, New Delhi 110 003 Phones:

EP–SLCM–June 2019 28

SECURITIES LAWS & CAPITAL MARKETS

Time allowed : 3 hours Maximum marks : 100

NOTE : Answer ALL Questions.

PART  I

Question 1

(a) A Mutual Fund having 300 units has shown Net Asset Value (NAV) of ̀ 8.75 and`9.45 at the beginning and at the end of the year respectively. The Mutual Fundhas given two options :

(i) Pay ̀ 0.75 per unit as dividend and ̀ 0.60 per unit as capital appreciation; or

(ii) These distributions are to be reinvested at an average NAV of `8.65 perunit

What difference it would make in terms of return available and which option ispreferable ?

(b) 'A stock exchange on its own can delist any security thereon'. Explain howRecognized Stock Exchange delists any securities listed thereon under SecuritiesContracts (Regulations) Rules, 1957.

(c) Your Board of directors is contemplating to take-up the agenda to issue ESOSin next meeting. Being a Company Secretary, advise your Board of directorsabout brief procedure for issuing of securities under SEBI Employees StockOption Scheme (ESOS) by a listed Company.

(d) Can a Company buy-back its own shares or any specified securities throughnegotiated deals or through any private arrangements ? Comment with methodsallowed for buy-back. (5 marks each)

Answer 1(a)

Basic data for Computation

Opening NAV Rs. 8.75

Closing NAV Rs. 9.45

Dividend Rs. 0.75

Capital gain appreciation [Closing NAV- Opening NAV] Rs. 0.70

Capital gain distribution Rs. 0.60

Price paid at the year beginning [300 units x Rs. 8.75] Rs. 2,62528

Page 31: EXECUTIVE PROGRAMME (New Syllabus) · GUIDELINE ANSWERS EXECUTIVE PROGRAMME (New Syllabus) JUNE 2019 MODULE 2 ICSI House, 22, Institutional Area, Lodi Road, New Delhi 110 003 Phones:

29 EP–SLCM–June 2019

Option I: Returns are distributed to Mutual Fund Holders

Total Fund Assets:

NAV on closing date [9.45 x 300] Rs. 2,835

Dividend payable [0.75 x 300] Rs. 225

Capital gain distribution [0.60 x 300] Rs. 180

Rs. 3,240

Returns : (Rs. 3, 240- Rs.2,625)/Rs. 2,625 = 23.43 %

OR

Alternate Answer

Option I: When Dividend and Capital Gain are paid:

Calculation of monthly return on the mutual funds:

r = ? ?1-t

tt1-t 1

NAVG+ l+NAV -NAV

Or, r = ? ?8.75 Rs.

0.60) Rs. + (Rs.0.75 + 8.75 Rs. - 9.45 Rs.

= 8.75

1.35 + 0.70= 23.43 %

Option II: The distributions are reinvested at an average NAV of Rs. 8.65 per unit

(i) Distribution reinvested

Capital gain [Rs. 0.60 x 300] Rs. 180

Dividend [Rs.0.75 x 300] Rs. 225

Total Distribution Rs. 405

No. of units [Total distributions ÷ Average NAV per unit= Rs. 405 ÷ Rs. 8.65]46.82

(ii) Preparation of fund position

NAV on closing date fund assets

- 300 [email protected] Rs. 2,835.00

- 46.82 units @Rs.9.45 Rs. 442.45

Rs. 3,277.45

Returns: (3,277.45-2,625)/ 2625 = 24.85 %

Page 32: EXECUTIVE PROGRAMME (New Syllabus) · GUIDELINE ANSWERS EXECUTIVE PROGRAMME (New Syllabus) JUNE 2019 MODULE 2 ICSI House, 22, Institutional Area, Lodi Road, New Delhi 110 003 Phones:

EP–SLCM–June 2019 30

OR

Alternate Answer

Option II: When Dividend and Capital Gain are reinvested:

If all dividends and capital gain are reinvested into additional units at Rs. 8.65 perunit the position would be.

Total amount reinvested = Rs. 1.35 x 300 = Rs.405

Additional units added = Rs. 8.65405

= 46.82 units or 47 units

Value of units at the end = 346.82 units x Rs. 9.45 = Rs. 3, 277.45

Or = 347 units x Rs. 9.45 = Rs.3, 279.15

Price paid for 300 units as at the beginning = (300 x Rs. 8.75) = Rs. 2,625

Return = (Rs. 3, 277.45 – Rs. 2, 625)/ Rs. 2, 625 = 24.86%

Or Return = 2,625 Rs.

625 2, Rs. - 279.15 3, Rs. =

2,625 Rs.654.15 Rs.

= 24.92%

Conclusion: From the above, it can be said that reinvestment option is better as ityield higher return by 1.49%.

Answer 1(b)

Rule 21 of the Securities Contracts (Regulations) Rules, 1957 deals with the delistingof securities. A recognized stock exchange may, without prejudice to any other actionthat may be taken under the Act or under any other law for the time being in force, delistany securities listed thereon on any of the following grounds in accordance with theregulations made by the Securities and Exchange Board of India, namely:—

(a) the company has incurred losses during the preceding three consecutive yearsand it has negative networth;

(b) trading in the securities of the company has remained suspended for a period ofmore than six months;

(c) the securities of the company have remained infrequently traded during thepreceding three years;

(d) the company or any of its promoters or any of its director has been convicted forfailure to comply with any of the provisions of the Act or the Securities andExchange Board of India Act, 1992 or the Depositories Act, 1996 or rules, regulations,agreements made thereunder, as the case may be and awarded a penalty of notless than rupees one crore or imprisonment of not less than three years;

(e) the addresses of the company or any of its promoter or any of its directors, are notknown or false addresses have been furnished or the company has changed itsregistered office in contravention of the provisions of the Companies Act, 2013;

(f) shareholding of the company held by the public has come below the minimum

Page 33: EXECUTIVE PROGRAMME (New Syllabus) · GUIDELINE ANSWERS EXECUTIVE PROGRAMME (New Syllabus) JUNE 2019 MODULE 2 ICSI House, 22, Institutional Area, Lodi Road, New Delhi 110 003 Phones:

31 EP–SLCM–June 2019

level applicable to the company as per the listing agreement under the Act andthe company has failed to raise public holding to the required level within the timespecified by the recognized stock exchange. No securities shall be delisted unlessthe company concerned has been given a reasonable opportunity of being heard.

Answer 1(c)

To

The Board of Directors

Sub: Procedure for issuing securities under SEBI (Share Based Employee Benefits)Regulations, 2014

As it has been decided by the Board of Directors (BoD), to issue employee stockoption, to take up as an agenda in the next meeting of the BoD, the following is theprocedure for issuing ESOP by a Listed Company:

• Hold a Board Meeting to consider and approve ESOP and formation ofCompensation Committee;

• Compensation committee shall plan draft the scheme of ESOP;• Hold Board meeting to adopt the final scheme, appoint a registered Merchant

banker and approve the notice of the General meeting for shareholders’ approvalby passing special resolution;

• Hold General Meeting for approval of shareholders;• Make an application to the stock exchange for obtaining in-principal approval for

listing of proposed ESOP shares;• Issue letter of grant of option to the eligible employees along with the letter of

acceptance of option;• If the grant is made to NRI employees, comply with norms of RBI as per FDI

Policy.• On receipt of letter of acceptance of option along with upfront payment (if any),

from the employee issue the option certificates;• There shall be a minimum vesting period of one year and after expiry of vesting

period, the company shall issue a letter of vesting along with the letter of exerciseof options;

• Receipt to letter of exercise from the employee;• Hold a Board Meeting at the suitable Interval during the exercise period for

allotment of shares on options exercised by the Employees;• Dispatch of letter of allotment along with the share certificates or credit the

shares so allotted with the Depositories;• Make an application to the Stock exchange for listing of the Shares so allotted; and• Receipt of listing approval from the Stock Exchange(s).

Mr XCompany Secretary

Note : Notice of Meetings (both Board & Shareholder) has to be given.For Board Meetings : atleast 7 days notice.For Shareholder meeting : atleast 21 days notice.

Page 34: EXECUTIVE PROGRAMME (New Syllabus) · GUIDELINE ANSWERS EXECUTIVE PROGRAMME (New Syllabus) JUNE 2019 MODULE 2 ICSI House, 22, Institutional Area, Lodi Road, New Delhi 110 003 Phones:

EP–SLCM–June 2019 32

Answer 1(d)

As per Regulation 4 (vi) of the SEBI (Buy-back of Securities) Regulations, 2018, acompany shall not buy-back its shares or other specified securities from any personthrough negotiated deals, whether on or off the stock exchange or through spot transactionsor through any private arrangement.

As per Regulation 4 (iv) of the SEBI (Buy-back of Securities) Regulations, 2018, acompany may buy-back its shares or other specified securities by any one of the followingmethods:

(a) from the existing shareholders or other specified securities holders on aproportionate basis through the tender offer;

(b) from the open market through —

(i) book-building process,

(ii) stock exchange;

(c) from odd-lot holders.

Attempt all parts of either Q. No. 2 or Q. No. 2A

Question 2

(a) A listed company can apply to stock exchange for re-classification of thePromoter's holdings as public shareholders under SEBI regulations. Whetherfollowing promoters can apply for re-classification with reference to SEBIregulations ?

(a) Promoter is declared as willful defaulter as per RBI guidelines.

(b) Promoter is holding 12% of total voting rights in the listed entity.

(c) Promoter is acting as CEO of the listed entity.

(d) The promoter company has outstanding listing fees only for one year.

(b) Explain the Modes of Payment to the shareholders of the Target Company onacquisition of shares by the acquirer under SEBI (Substantial Acquisition ofShares and Takeovers) Regulations, 2011.

(c) An acquirer, holding 25% or more but less than maximum permissible non-public shareholding of the Target Company can acquire such additional sharesas would entitle him to exercise more than 5% of the voting rights in any financialyear. Explain the statement indicating the creeping acquision limit for makingan open offer by an acquirer.

(d) SEBI has been given necessary autonomy and authority to regulate and developan orderly market. Elucidate the statement in the light of statutory powers vestedwith SEBI.

(e) Life-Changing Assets Management Ltd., a mutual funds company desires toengage a bollywood celebrity to popularize its schemes. Explain the SEBIprovisions with regard to celebrity endorsements of Mutual Funds at industrylevel.

Page 35: EXECUTIVE PROGRAMME (New Syllabus) · GUIDELINE ANSWERS EXECUTIVE PROGRAMME (New Syllabus) JUNE 2019 MODULE 2 ICSI House, 22, Institutional Area, Lodi Road, New Delhi 110 003 Phones:

33 EP–SLCM–June 2019

OR (Alternate question to Q. No. 2)

Question 2A

(i) M/s Highspeed Ltd. manufacturing a car components for leading car manufacturer.Its public issue of `500 crore was fully subscribed. The public issue moneyought to be utilized for setup an assembly-line for the existing business. Out of`500 crore, the company spent ̀ 400 crore for assembly-line. The managementconsultant, hired for Business Process re-engineering has suggested to investbalance amount to setup bike components manufacturing unit. You, beingcompany secretary of the company, advise on the opinion of managementconsultant by referring provisions of SEBI Guidelines. (4 marks)

(ii) From the following information, calculate the Enterprise Value of E Ltd. :

Balance Sheet of E Ltd. as on 31st March, 2018

Liabilities Amount Assets Amount(` Lakh) (` Lakh)

Share Capital (Face Value Non-Current Assets 2,550`2) 952 Current Assets :

Reserves & Surplus 48 Cash & Cash Equivalent 102

Minority Interest 115 Other Current Assets 1,323

Short-term Borrowings 2,860

3,975 3,975

Current Market Price Per Share is ̀ 96. (4 marks)

(iii) The Board of directors of a listed company desires to delist its equity sharesfrom all the recognised stock exchanges. The voting details through postalballot are as under :

— Total nos. of voters : 7,000 (Public : 5,000 & Promoters : 2,000)

— Voting at shareholders meeting :

(a) Public shareholders :

In favour : 3,300 votes

In against : 1,700 votes

(b) All promoters shareholders have voted in favour of resolution.

By referring SEBI delisting regulation, decide upon the resolution passed by theshareholders. (4 marks)

(iv) The financial data of a listed company as on 31st March, 2018 are as follows :

Authorized equity share capital `10 crore

(1 crore shares of ̀ 10 each)

Page 36: EXECUTIVE PROGRAMME (New Syllabus) · GUIDELINE ANSWERS EXECUTIVE PROGRAMME (New Syllabus) JUNE 2019 MODULE 2 ICSI House, 22, Institutional Area, Lodi Road, New Delhi 110 003 Phones:

EP–SLCM–June 2019 34

Paid-up equity share capital ` 5 crore

General reserve ` 3 crore

Debenture redemption reserve ` 2 crore

The Board of directors of your company passed resolution by circulation forbuy-back of shares to the extent of 9% of the company's paid-up share capitaland free reserves. You are required to examine the validity of the proposal withreference to the provisions of the SEBI Regulations. (4 marks)

(v) Z Ltd. has issued Sweat Equity Shares for a non-cash consideration. What arethe possible accounting treatments in the books of Z Ltd. ? (4 marks)

Answer 2(a)

Regulation 31A of the SEBI (Listing Obligations and Disclosure Requirements)Regulations, 2015 (“SEBI LODR Regulations”) deals with the conditions for re-classificationof any person as promoter / public.

Based on the provisions of Regulation 31A of the SEBI LODR Regulations, thefollowing are the answers to the circumstances given:

(a) Promoter is declared as wilful defaulter as per RBI guidelines. As per Regulation31A (b)(vi), the Promoter declared as a ‘wilful defaulter’ as per the ReserveBank of India Guidelines is not eligible to apply for re-classification themselvesas public shareholders.

(b) Promoter is holding 12 % of total voting rights in the listed entity. As perRegulation 31A (b)(i), if the promoter holds more than 10% of the total votingrights in the listed entity, they are not eligible for re-classification. Hence in thiscase, the promoter is not eligible to apply for re-classification.

(c) As per Regulation 31A (b)(v), if the promoter acts as a key managerial person inthe listed entity he is not eligible for re-classification. In the given case, thepromoter is a CEO hence he is not eligible for re-classification

(d) As per Regulation 31A (c)(iii), the listed entity shall not have any outstandingdues to the SEBI, the stock exchanges or the depositories to be is eligible toapply for reclassification. In this case, the promoter company has outstandinglisting fees for one year, hence it is not eligible for re-classification.

Answer 2(b)

As per regulation 9 of the SEBI (Substantial Acquisition of Shares and Takeovers)Regulations, 2011, the offer price may be paid, —

(a) in cash;

(b) by issue, exchange or transfer of listed shares in the equity share capital of theacquirer or of any person acting in concert;

(c) by issue, exchange or transfer of listed secured debt instruments issued by theacquirer or any person acting in concert with a rating not inferior to investmentgrade as rated by a credit rating agency registered with the SEBI;

Page 37: EXECUTIVE PROGRAMME (New Syllabus) · GUIDELINE ANSWERS EXECUTIVE PROGRAMME (New Syllabus) JUNE 2019 MODULE 2 ICSI House, 22, Institutional Area, Lodi Road, New Delhi 110 003 Phones:

35 EP–SLCM–June 2019

(d) by issue, exchange or transfer of convertible debt securities entitling the holderthereof to acquire listed shares in the equity share capital of the acquirer or ofany person acting in concert; or

(e) a combination of the mode of payment of consideration stated in clause (a),clause (b), clause (c) and clause (d).

Answer 2(c)

As per Regulation 3 (2) of the SEBI (Substantial Acquisition of Shares and Takeovers)Regulations, 2011, an acquirer who holds 25% or more but less than maximum permissiblenon-public shareholding of the Target Company, can acquire such additional shares aswould entitle him to exercise more than 5% of the voting rights in any financial yearending March 31 only after making a Public Announcement to acquire minimum twentysix percent shares of Target Company from the shareholders through an Open Offer.

— An acquirer can, who has reached at a level of 25% or more and made detailedpublic statement to acquire = 26% shares, received in this process.

— Then acquirer in any financial year cannot take > 5% share.

— If acquirer wants to acquire > 5% share in one FY, again public offer shall bemade to receive 26% (or) more shares subject to delisting level.

Answer 2(d)

With regulatory jurisdiction extended over the corporates with respect to issue ofcapital and transfer of securities, in addition to all intermediaries and persons associatedwith securities market, SEBI is vested with following statutory powers:

1. protecting the interests of investors in securities,

2. promoting the development of the securities market, and

3. regulating the securities market.

Page 38: EXECUTIVE PROGRAMME (New Syllabus) · GUIDELINE ANSWERS EXECUTIVE PROGRAMME (New Syllabus) JUNE 2019 MODULE 2 ICSI House, 22, Institutional Area, Lodi Road, New Delhi 110 003 Phones:

EP–SLCM–June 2019 36

Its regulatory jurisdiction extends over corporates in the issuance of capital andtransfer of securities, in addition to all intermediaries and persons associated withsecurities market. SEBI can:

— specify the matters to be disclosed and the standards of disclosure required forthe protection of investors in respect of issues;

— issue directions to all intermediaries and other persons associated with thesecurities market in the interest of investors or of orderly development forsecurities market; and

— conduct enquiries, audits and inspection of all concerned and adjudicate offencesunder the Act.

In short, it has been given necessary autonomy and authority to regulate and developan orderly securities market.

Answer 2(e)

SEBI vide its Circular No. CIR/lMD/DF/23/2017 dated 15/03/2017 reviewed theadvertisement guidelines for mutual funds.

In this respect, it has been decided to permit celebrity endorsements at industrylevel, for the purpose of increasing awareness of Mutual Funds as a financial productcategory. However, such celebrity endorsements of Mutual Funds at industry level,shall be subject to the following conditions:

i. Celebrity endorsement shall be allowed only at the industry level, for the purposeof increasing awareness of Mutual Funds as a financial product category. Suchcelebrity endorsements should not promote a scheme of a particular MutualFund or be used as a branding exercise of a Mutual Fund house.

ii. Expenses towards such celebrity endorsements for increasing awareness of MutualFunds shall be limited to the amounts that are aggregated by Mutual Funds atindustry level for the purpose of conducting investor education and awarenessinitiatives, in terms of clause F of SEBI circular dated September 13, 2012.

iii. Prior approval of SEBI shall be required for issuance of any endorsement ofMutual Funds as a financial product, which features a celebrity for the purposeof increasing awareness of Mutual Funds.

Answer 2A(i)

According to the Regulation 59 of the SEBI (ICDR) Regulations, 2018, the promoters,or shareholders in control of an issuer, shall provide an exit offer to dissenting shareholdersas provided for in the Companies Act, 2013, in case of change in objects or variation inthe terms of contract related to objects referred to in the offer document as per conditionsand manner as provided in Schedule XX of the SEBI (ICDR) Regulations, 2018.

According to the Schedule XX, the promoter or the shareholders in control, as thecase may be, shall make an exit offer in accordance with the provisions of Chapter II ofSEBI (ICDR) Regulations to the dissenting shareholders, in cases only if a public issuehas opened after April 1, 2014; if:

(a) The proposal for change in objects or variation in terms of a contract, referred to

Page 39: EXECUTIVE PROGRAMME (New Syllabus) · GUIDELINE ANSWERS EXECUTIVE PROGRAMME (New Syllabus) JUNE 2019 MODULE 2 ICSI House, 22, Institutional Area, Lodi Road, New Delhi 110 003 Phones:

37 EP–SLCM–June 2019

in the offer document is dissented by atleast ten percent of the shareholderswho voted in the general meeting; and

(b) The amount to be utilized for the objects for which the offer document wasissued is less than seventy five percent of the amount raised (including theamount for general corporate purposes as disclosed in the offer document).

In the given problem M/s Highspeed Ltd. has utilized Rs. 400 crore out of totalRs. 500 crore raised through public issue. As the amount utilized is more than75% of the total amount raised, the company may utilize the remaining unutilized100 crore for the purpose as stated in the question.

Answer 2A(ii)

Enterprise Value = Market Capitalisation + Debt + Minority Interest andPreferred Shares - Total Cash and Cash Equivalent

No. of shares = 952 Lakh/ 2 = 476 Lakh

Enterprise Value = 96 x 476 Lakhs + 115 Lakhs +2,860 Lakhs -102 Lakhs =Rs. 48,569 Lakhs

Answer 2A(iii)

Regulation 6 of the SEBI (Delisting of Equity Shares) Regulations, 2009, prescribedthat if a company proposes to delist its equity shares from all the recognized stockexchanges where they are listed, it is required to obtain the prior approval of shareholdersof the company by special resolution passed through postal ballot, after disclosure of allmaterial facts in the explanatory statement sent to the shareholders in relation to suchresolution. However, the special resolution shall be acted upon if and only if the votescast by public shareholders in favour of the proposal amount to at least two times thenumber of votes cast by public shareholders against it.

In the mentioned case, the resolution was passed as special resolution as it hasapproval of 75% of the shareholders, however, the resolution does not have requisiteapproval of public shareholders. The votes cast in favour of resolution (3300 votes) isnot the twice of the votes cast against the resolution (1700 votes) by the publicshareholders. Therefore, the special resolution cannot be acted upon by the company.

Answer 2A(iv)

The financial data of a listed company as on 31st March, 2018 are as follows:

Authorized equity share capital Rs. 10 crores

(1 crore shares of Rs. 10 each)

Paid- up equity share capital Rs. 5 crores

General reserve Rs. 3 crores

Debenture redemption reserve Rs. 2 crores

According to the regulation 5 of the SEBI (Buy-back of Securities) Regulations,2018, the company shall not authorize any buy-back (whether by way of tender offer or

Page 40: EXECUTIVE PROGRAMME (New Syllabus) · GUIDELINE ANSWERS EXECUTIVE PROGRAMME (New Syllabus) JUNE 2019 MODULE 2 ICSI House, 22, Institutional Area, Lodi Road, New Delhi 110 003 Phones:

EP–SLCM–June 2019 38

from open market or odd lot) unless a special resolution has been passed at a generalmeeting of the company authorising the buy-back. However, special resolution is notrequired, where the buy-back is, ten per cent or less of the total paid-up equity capitaland free reserves of the company; and such buy-back has been authorised by the boardof directors by means of a resolution passed at its meeting.

In the given case, the company desired to buy-back of shares to the extent of 9% ofpaid-up capital and free reserves by way of passing of board resolution through circulation,however as per above regulations, the board resolution should be passed at its meetingnot through circulation. Therefore with reference to the above stated provisions, theproposal of buy-back is not valid.

Answer 2A(v)

As per regulation 9 of the SEBI (Issue of Sweat Equity) Regulations, 2002, wherethe sweat equity shares are issued for a non-cash consideration, such non-cashconsideration shall be treated in the following manner in the books of account of thecompany: ­

1. where the non-cash consideration takes the form of a depreciable or amortizableasset, it shall be carried to the balance sheet of the company in accordancewith the relevant accounting standards; or

2. where the above clause is not applicable, it shall be treated as expense asprovided in the relevant accounting standards.

Question 3

(a) A listed NBFC has been granted licence to run as small finance bank by theReserve Bank of India under recently announced policy to improve the financialinclusion of the country. During the last three years, the attrition rate for toplevel management employees was not too high As, RBI has granted licences tomany small banks, therefore, the promoters of the Bank feels that attrition ratewill be high in coming . period. The Board of directors wishes to allot SweatEquity shares to employees. You, being compliance officer of the Bank, advisethe Board about pricing of the Sweat Equity shares.

(b) An unlisted public company ("Acquirer") doing business of exporting steel and itis a part of the Promoter Group of Maurya Hotels (India) Ltd. (MHIL), a companylisted on stock exchange. In view of improving its efficiency, MHIL is planningto restructure its group. The Acquirer has agreed to enter into a scheme ofarrangement where the shares held by the promoter group companies (eightcompanies) will be transferred to it. Post-merger, the shareholding of the Acquirerin the Company will increase from 2% to 24%. However, the overall promotershareholding will remain unchanged. You, being practicing company secretary,appointed as consultant by the Acquirer, answer the following :

(i) Will the transfer of shares trigger an obligation to make an open offer underthe SEBI (SAST) Regulations on the Acquirer ?

(ii) What are the disclosure requirements under the SAST Regulations, if any,that the parties to the scheme will have to comply with ?

Page 41: EXECUTIVE PROGRAMME (New Syllabus) · GUIDELINE ANSWERS EXECUTIVE PROGRAMME (New Syllabus) JUNE 2019 MODULE 2 ICSI House, 22, Institutional Area, Lodi Road, New Delhi 110 003 Phones:

39 EP–SLCM–June 2019

(c) Explain what is Informal Guidance under SEBI (Informal Guidance) Scheme,2013 and who can seek guidance from SEBI ? (5 marks each)

Answer 3(a)

To

The Board of Directors

Sub: Pricing of the Sweat Equity Shares

As the Board of the Directors has decided to allot sweat equity shares to employees,the following are the provisions relating to pricing of sweat equity shares:

1. According to the regulation 7 of the SEBI (Issue of Sweat Equity) Regulations,2002, the price of sweat equity shares shall not be less than the higher of thefollowing:

(a) the average of the weekly high and low of the closing prices of the relatedequity shares during last six months preceding the relevant date; or

(b) the average of the weekly high and low of the closing prices of the relatedequity shares during the two weeks preceding the relevant date.

2. If the shares are listed on more than one stock exchange, but quoted only onone stock exchange on given date, then the price on the stock exchange shallbe considered.

3. If the share price is quoted on more than one stock exchange, then the stockexchange where there is highest trading volume during that date shall beconsidered.

4. If the shares are not quoted on the given date, then the share price on the nexttrading day shall be considered.

Answer 3(b)

(i) Regulations 3, 4 and 5 of the SEBI (Substantial Acquisition of Shares andTakeovers) Regulations, 2011 set out the events that trigger an obligation tomake an open offer on the acquirer (along with persons acting in concert). Thesaid triggers points are as follows:

• Acquisition (directly or indirectly) of such shares or voting rights in an Indianlisted company whereby the acquirer becomes entitled to exercise 25% ormore of the voting rights in such Indian listed company;

• Where an acquirer (along with persons acting in concert with him) is alreadyentitled to exercise 25% or more of the voting rights in an Indian listedcompany, and acquires (directly or indirectly) additional shares or votingrights entitling an acquirer to exercise more than 5% voting rights in anIndian listed company, in a financial year; and

• Acquisition (directly or indirectly) of control.

Since the Acquirer, in the above facts, does not attract any of the triggers set

Page 42: EXECUTIVE PROGRAMME (New Syllabus) · GUIDELINE ANSWERS EXECUTIVE PROGRAMME (New Syllabus) JUNE 2019 MODULE 2 ICSI House, 22, Institutional Area, Lodi Road, New Delhi 110 003 Phones:

EP–SLCM–June 2019 40

out above, the transfer of shares will not impose any obligation on the Acquirerto make an open offer under the SEBI (SAST) Regulations.

(ii) The Acquirer and promoter group companies will be required to make a disclosureof change in shareholding under Regulation 29(2) of the SEBI (SubstantialAcquisition of Shares and Takeovers) Regulations, 2011. According to Regulation29(3) of the said regulations, the disclosure should be made within 2 workingdays of such acquisition or the disposal to the target company at its registeredoffice and to the stock exchanges where the shares of the target company arelisted.

Answer 3(c)

In the interests of better regulation of and orderly development of the securitiesmarket, SEBI has issued SEBI (Informal Guidance) Scheme 2003. The following personsmay make a request for informal Guidance under the scheme:

(a) any intermediary registered with the SEBI.

(b) any listed company.

(c) any company which intends to get any of its securities listed and which hasfiled either a listing application with any stock exchange or a draft offer documentwith the SEBI or the Central Listing authority.

(d) any mutual fund trustee company or asset management company.

(e) any acquirer or prospective acquirer under the SEBI (Substantial Acquisition ofShares and Takeovers) Regulations, 1997. (Now SEBI Takeover Regulations, 2011)

The Guidance Scheme, further deals with various aspects such as the nature ofrequest, fees to be accompanied along with request letter, disposal of requests, SEBI’sdiscretion not to respond certain types of requests and confidentiality of requests, etc.

Question 4

(a) The price of equity share of a listed company viz. NextDial Ltd. (NDL) increasedfrom `10 to high of `50 i.e. a rise of 500% during the period 1st April, 2018 to30th Sept., 2018. NDL had entered into a Share Purchase Agreement (SPA)with the proposed acquirer(s) to acquire 40% of the subscribed equity sharecapital as of 31st Aug., 2018 which would result in change of management. Thisinitial discussion on the deal was made on 1st April, 2018 but SPA was signedon 25th April, 2018. During 1st April, 2018 to 30th Sept., 2018, the promoter andhis wife dealt in the script of NextDial Ltd. Referring to the provisions of SEBI(PIT) Regulations, answer the following :

(i) Define Unpublished Price Sensitive Information.

(ii) Whether there was any Unpublished Price Sensitive Information (UPSI) ?

(iii) What will be the date of UPSI ?

(iv) What are the factors to be taken into account by the adjudicating officerwhile imposing penalty for the act ? (8 marks)

Page 43: EXECUTIVE PROGRAMME (New Syllabus) · GUIDELINE ANSWERS EXECUTIVE PROGRAMME (New Syllabus) JUNE 2019 MODULE 2 ICSI House, 22, Institutional Area, Lodi Road, New Delhi 110 003 Phones:

41 EP–SLCM–June 2019

(b) The financial data of Natural Energy Limited as on 31st March, 2018 are asunder :(i) Authorised Share Capital : ̀ 700 crore(ii) Paid-up Capital : ̀ 300 crore(iii) Free Reserves : ̀ 800 croreThe company has pending convertible debenture of ̀ 150 crore, due for conversionin financial year 2018-19. The company proposes to issue bonus shares in theratio of 1 : 1 after conversion of debenture. You being a company secretary,advise on the procedure to be followed by referring SEBI regulations.

(7 marks)

Answer 4(a)

(i) The SEBI (Prohibition of Insider Trading) Regulations, 2015 defines “UnpublishedPrice Sensitive Information” which means any information, relating to a companyor its securities, directly or indirectly, that is not generally available which uponbecoming generally available, is likely to materially affect the price of thesecurities and shall, ordinarily including but not restricted to, information relatingto the following:-

(i) financial results;

(ii) dividends;

(iii) change in capital structure;

(iv) mergers, de-mergers, acquisitions, delistings, disposals and expansion ofbusiness and such other transactions;

(v) changes in key managerial personnel.

(ii) The promoter of Next Dial Ltd. was involved in the negotiation for fixing theshare price and traded in the shares of the company while being privy to theprice sensitive information. Therefore, it is alleged that promoter had dealt in theshares of the company while in possession of or on the basis of the UnpublishedPrice Sensitive Information (UPSI). Additionally, he has also communicated orcounseled, directly or indirectly, the UPSI to his wife, who in turn has alsotraded in the shares of the company.

The definition of Price Sensitive Information also includes the following as PriceSensitive Information; (a) amalgamation, mergers or takeovers, (b) disposal ofthe whole or substantial part of the undertaking. So Unpublished Price SensitiveInformation (UPSI) was existed.

(iii) In the mentioned case, Share Purchase Agreement (SPA) was signed on 25thApril, 2018. Taking into consideration when the SPA was signed, the UPSI isconsidered to have originated on April 25, 2018.

(iv) While imposing monetary penalty, it is obligatory to consider the factors stipulatedin Section 15J of the SEBI Act, 1992 which reads as under:

a. the amount of disproportionate gain or unfair advantage, wheneverquantifiable, made as a result of the default;

Page 44: EXECUTIVE PROGRAMME (New Syllabus) · GUIDELINE ANSWERS EXECUTIVE PROGRAMME (New Syllabus) JUNE 2019 MODULE 2 ICSI House, 22, Institutional Area, Lodi Road, New Delhi 110 003 Phones:

EP–SLCM–June 2019 42

b. the amount of loss caused to an investor or group of investors as a result ofthe default;

c. the repetitive nature of the default.

Answer 4(b)

Chapter XI consisting of regulation 293-295 of the SEBI (ICDR) Regulations, 2018stipulates the provisions with respect to issue of bonus shares. Regulation 293 providesthat, subject to the provisions of the Companies Act, 2013 or any other applicable law,a listed issuer shall be eligible to issue bonus shares to its members if:

(i) it is authorized by its Articles of Association for issue of bonus shares,capitalization of reserves, etc;

Provided that if there is no such provision in the Articles of Association, theissuer shall pass a resolution at its general body meeting making provisions inthe Articles of Association for capitalization of reserve;

(ii) it has not defaulted payment of interest or principal in respect of fixed depositsor debt securities issued by it;

(iii) it has not defaulted in payment of statutory dues of the employees such ascontribution to provident fund, gratuity and bonus;

(iv) any outstanding partly paid shares on the date of the allotment of the bonusshares are made fully paid-up;

(v) any of its promoters or directors is not a fugitive economic offender.

The Article of Association must authorize it to issue the bonus shares. If there is noprovision in the Article for Bonus shares, firstly articles shall be amended by the company.

Determination of increase in authorized capital required:

a. Paid-up share capital as on 31/03/2018 : Rs. 300 crores

b. Paid-up share capital after conversion of debentures : Rs. 450 crores

c. Proposed Bonus issue : One share forevery share held

d. Post Bonus issue Capital : Rs. 900 Crores

Since authorized capital is Rs.700 crores only, it is required to increase the authorizedcapital by 200 crores or more.

The Reserves & Surplus is Rs. 800 crores, therefore bonus issue of Rs. 450 crorescan be made out of reserves & surplus.

Other conditions to be followed:

• Bonus shares shall not be issued in lieu of dividend

• A resolution shall be passed by the Board in its meeting

• Bonus issue shall be completed within 15 days from the date of approval ofBoard of Directors, if shareholders’ approval is not required

Page 45: EXECUTIVE PROGRAMME (New Syllabus) · GUIDELINE ANSWERS EXECUTIVE PROGRAMME (New Syllabus) JUNE 2019 MODULE 2 ICSI House, 22, Institutional Area, Lodi Road, New Delhi 110 003 Phones:

43 EP–SLCM–June 2019

• Where approval of shareholders is required, bonus issue shall be completedwithin 2 months from the date of meeting of Board of Directors wherein thedecision to announce the bonus issue was taken subject to shareholder’s approval

• A bonus issue, once announced, shall not be withdrawn.

PART II

Question 5

(a) What are the Option contracts ? You are required to compute the profit/loss foreach investors in below option contracts :(i) Mr. X writes a call option to purchase share at an exercise price of `60 for

a premium of `12 per share. The share price rises to `62 by the time theoption expires.

(ii) Mr. Y buys a put option at an exercise price of `80 for a premium of `8.50per share. The share price falls to `60 by the time the option expires.

(iii) Mr. Z writes a put option at an exercise price of ̀ 80 for a premium of ̀ 11 pershare. The price of the share rises to `96 by the time the option expires.

(iv) Mr. XY writes a put option with an exercise price of ̀ 70 for a premium of ̀ 8per share. The price falls to `48 by the time the option expires.

(b) What do you mean by 'Research Analysts' ? Elucidate the net worth requirements,and role and responsibilities of Research Analyst as per SEBI (Research Analyst)Regulations, 2014.

(c) "Prior information of open position of any share during market hours can easilyfluctuate the price of the share". How Preventive Surveillance helps to reducethe fraudulent price variation in the shares in a day ? (5 marks each)

Answer 5(a)

Options Contract give its holder the right, but not the obligation, take or make deliveryon or before a specified date at a stated price.

Option Contracts are classified into two types on the basis of which party has theoption:

a. Call Option – A call option is with the buyer and gives the right to take delivery.The buyer of the call option has a right to buy the underlying asset from theoption seller.

b. Put Option – The put option is with the seller and the option gives the right totake delivery. The buyer of the put option has a right to sell the underlying assetto the option seller.

(i) For Mr. X : Profit = [{Rs. 60 – Rs. 62} + Rs. 12] = Rs. 10

(ii) For Mr. Y : Profit = [{Rs. 80 – Rs. 60} - Rs. 8.50] = Rs. 11.50

(iii) For Mr. Z : Profit = Rs. 11.00 (Option will not be exercised)

(iv) For Mr. XY : Loss = [{Rs. -70 + Rs. 48} + Rs. 8] = - Rs. 14

Page 46: EXECUTIVE PROGRAMME (New Syllabus) · GUIDELINE ANSWERS EXECUTIVE PROGRAMME (New Syllabus) JUNE 2019 MODULE 2 ICSI House, 22, Institutional Area, Lodi Road, New Delhi 110 003 Phones:

EP–SLCM–June 2019 44

Answer 5(b)

As per the SEBI (Research Analysts) Regulations, 2014, “Research Analyst” meansa person who is primarily responsible for:

i. preparation or publication of the content of the research report; or

ii. Providing research report; or

iii. making ‘buy / sell / hold recommendation; or

iv. giving price target; or

v. offering an opinion concerning public offer, with respect to securities that arelisted or to be listed in a stock exchange, whether or not any such person hasthe job title of ‘Research Analyst’ and include any other entities engaged inissuance of Research Report or Research Analysis.

Net worth requirement : Body Corporate or Limited Liability Partnership firm – notless than Rs. 25 lakhs; Individual or Partnership firm shall have net tangible assets ofvalue not less than Rs. 1 Lakh.

Role and Responsibility : They study companies and Industries, analyse raw dataand make forecast or recommendations about whether to buy, hold or sell securities.They analyse information to provide recommendations about investments in securitiesto their clients. Investors often view analysts as experts and important source ofinformation about the securities they review and often rely on their advice. There arebasically three broad types of analysts, viz. sell-side analysts, buy-side analysts andindependent analysts.

Answer 5(c)

Market Surveillance plays a vital role in ensuring market integrity which is coreobjective of regulators. Market integrity is achieved through combination of surveillance,inspection, investigation and enforcement of relevant laws and rules. Globally MarketSurveillance is either conducted by the regulators or Exchanges or both. In India, theprimary responsibility of market surveillance has been entrusted to Stock exchangesand is being closely monitored by SEBI. Market Surveillance is broadly categorized intwo parts, viz. Preventive Surveillance and Post Trade Surveillance.

Preventive Surveillance

— Stringent on-boarding norms for Trading members – Stringent net worth, background, viability, etc. checks while on boarding Trading members.

— Index Circuit Filters – It brings coordinated trading halt in all equity and equityderivative markets at 3 stages of the index movement, either way viz. at 10%,15% and 20% based on previous day closing index value.

— Trade Execution range – Orders are matched and trades take place only if thetrade price is within the reference price and execution range.

— Order Value Limitation – Maximum Order Value limit allowed per order.

— Cancel on Logout – All outstanding orders are cancelled, if the enabled userlogs out.

Page 47: EXECUTIVE PROGRAMME (New Syllabus) · GUIDELINE ANSWERS EXECUTIVE PROGRAMME (New Syllabus) JUNE 2019 MODULE 2 ICSI House, 22, Institutional Area, Lodi Road, New Delhi 110 003 Phones:

45 EP–SLCM–June 2019

— Kill Switch – All outstanding orders of the trading member are cancelled if tradingmembers executes kill switch.

— Risk Reduction Mode – Limits beyond which order level risk management shallbe initiated instead of trade level.

— Compulsory Close out – Incoming order, if it results in members crossing themargins available with the exchange, such order will be partially or fully cancelled,as the case may be, and further disallow the trading member to create freshpositions.

— Capital Adequacy Check – Refers to monitoring of trading member’s performanceand track record, stringent margin requirements, position limits based on capital,online monitoring of member positions and automatic disablement from tradingwhen limits are breached.

— Fixed Price Band / Dynamic Price band – Limits applied within which securitiesshall move; so that volatility is curbed, orderliness in brought about. For non-derivative securities, price band is 5%, 10% & 20%. For derivative products asoperating range of 10% is set and subsequently flexed based on marketconditions.

— Trade for Trade Settlement – The settlement of scrip’s available in this segmentis done on a trade for trade basis and netting off is allowed.

— Periodic Call Auction – Shifting the security form continuous to call auctionmethod.

— Rumour Verification – Any unannounced news about listed companies is trackedon online basis and letter seeking clarification is sent to the companies and thereply received is disseminated.

Attempt all parts of either Q. No. 6 or Q. No. 6A

Question 6

Write short notes on the following :

(a) Custodian of Securities(b) Key difference between WPI & CPI(c) Basis of SENSEX(d) High Net Worth Individuals(e) Bulk Deal. (3 marks each)

OR (Alternate question to Q. No. 6)

Question 6A

(i) "An Alternative Investment Fund which has been granted registration under aparticular category cannot change its category subsequent to registration, exceptwith the approval of the SEBI". Enumerate the conditions for approval of SEBI.

(ii) Dhruv has purchased 1000 shares @ ̀ 80 per share of a company. He wanted topay `5,000 in cash and balance through bank transfer to stock broker. As aCompany Secretary advise Dhruv by referring SEBI regulation/circular.

Page 48: EXECUTIVE PROGRAMME (New Syllabus) · GUIDELINE ANSWERS EXECUTIVE PROGRAMME (New Syllabus) JUNE 2019 MODULE 2 ICSI House, 22, Institutional Area, Lodi Road, New Delhi 110 003 Phones:

EP–SLCM–June 2019 46

(iii) "SEBI has amended the provisions related to registration of Sub-Broker to actas a market intermediary". Elucidate the statement and discuss the migrationpath for existing registered Sub-Brokers. (5 marks each)

Answer 6(a)

As per the SEBI (Custodian) Regulations, 1996, a custodian is a person who carrieson the business of providing custodial service to the client. The custodian keeps thecustody of the securities of the client. The custodian also provides incidental servicessuch as maintaining the accounts of securities of the client, collecting benefits or rightsaccruing to the client in respect to the securities.

Roles and Responsibilities:

• Administrate an protect the assets of the clients

• Open a separate custody account and deposit account in the name of eachclient

• Record Assets

• Conduct registration of Securities

Net Worth: Minimum Rs. 50 crore.

Answer 6(b)

Key difference between Wholesale Price Index (WPI) & Consumer Price Index (CPI)

i. Primary use of WPI is to have inflationary trend in the economy as a whole.However, CPI is used for adjusting income and expenditure streams for changesin the cost of living.

ii. WPI is based on wholesale prices for primary articles, administered process forfuel items and ex-factory prices for manufactured products. On the other hand,CPI is based on retail prices, which include all distribution costs and taxes.

iii. Prices of WPI are collected on voluntary basis while price data for CPI is collectedby Investigators by visiting markets.

iv. CPI covers only consumer goods and consumer services while WPI covers allgoods including intermediate goods transacted in the economy.

v. WPI weights primarily based on national accounts and enterprise survey dataand CPI weights are derived from consumer expenditure survey data.

Answer 6(c)

Sensitive Index or Sensex is the stock market index for the BSE. It is also sometimesreferred to as BSE S&P Sensex. The calculation of Sensex is done by a Free-Floatmethod that came into existence from September 1, 2003. The level of Sensex is directindication of the performance of 30 stocks in the market. The free-float method takesinto account the proportion of the shares that can be readily traded in the market. Thisdoes not include the ones held by various shareholders and promoters or other locked-shares not available in market.

Page 49: EXECUTIVE PROGRAMME (New Syllabus) · GUIDELINE ANSWERS EXECUTIVE PROGRAMME (New Syllabus) JUNE 2019 MODULE 2 ICSI House, 22, Institutional Area, Lodi Road, New Delhi 110 003 Phones:

47 EP–SLCM–June 2019

Steps to calculate Sensex

— The market capitalization is taken into account. This is done by multiplying allthe shares issued by the company with the price of its stock.

— BSE determines a Free-Float factor that is a multiple of the market capitalizationof the company. This helps in determining the free-float factor marketcapitalization based on details submitted by the company.

— Ratio and Proportion are used based on the base index of 100. This helps todetermine the Sensex.

Answer 6(d)

HNIs or High Net worth Individuals is a class of individuals who are distinguishedfrom other retail segment based on their net wealth, assets and investible surplus. Whilethere is no standard put forth for the classification, the definition of HNIs varies with thegeographical area as well as financial markets and institutions.

Though there is no specific definition, generally in the Indian context, individualswith over Rs. 2 crore investible surplus may be considered to be HNIs while those withinvestible wealth in the range of Rs. 25 lakhs to Rs. 2 crore may be deemed as EmergingHNIs.

If one is applying for an IPO of the equity shares in an Indian Company, generally,if one apply for amounts in excess of Rs. 2 lakhs, one falls under the HNI category. Onthe other hand, if one apply for amount under Rs. 2 lakhs, one is considered as a retailinvestor.

Answer 6(e)

Bulk deal is a trade, where total quantity bought or sold is more than 0.5% of thenumber of equity shares of a listed company.

Bulk deal can be transacted by the normal trading window provided by brokersthroughout the trading hours in a day. Bulk deals are market driven and take placethroughout the trading day.

The stock broker, who facilitates the trade, is required to reveal to the Stock Exchangeabout the bulk deals on daily basis.

Bulk orders are visible to everyone. If the bulk deal happens through a single trade,it should be notified to the Stock Exchange immediately upon the execution of the order.If it happens through multiple trades, it should be notified to the exchange within onehour from the closure of the trading.

Answer 6A(i)

As per SEBI Circular No. CIR/IMD/DF/12/2013 dated 7th August, 2013, only AIFswho have not made any investments under the category in which they were registeredearlier shall be allowed to make application for change in category. Such AIFs arerequired to make an application in Form A along with necessary supporting documents.Application fees of Rs. 1, 00,000 must be paid along with the application to SEBI. AIFsare not required to pay registration fees for such applications.

Page 50: EXECUTIVE PROGRAMME (New Syllabus) · GUIDELINE ANSWERS EXECUTIVE PROGRAMME (New Syllabus) JUNE 2019 MODULE 2 ICSI House, 22, Institutional Area, Lodi Road, New Delhi 110 003 Phones:

EP–SLCM–June 2019 48

If the AIF has received commitments/ raised funds prior to application for change incategory, the AIF shall be required to send letters/ emails to all its investors providingthem the option to withdraw their commitments/ fund raised without any penalties/ charges.Any fees collected from investors seeking to withdraw commitments/ funds shall bereturned to them. Partial withdrawal may be allowed subject to compliance with theminimum investment amount required under the AIF regulations.

The AIF shall not make any investments till deployment of fund as per the schemeother than in liquid funds/ bank deposits until approval for change in category is grantedby SEBI.

On approval of the request from SEBI, the AIF is required to send a copy of therevised placement memorandum and other relevant information to all its investors.

Answer 6A(ii)

SEBI vide its circular no. SEBI/HO/MIRSD/DoP/CIR/P/2018/113 dated 12/07/2018has discontinued acceptance of cash by Stock Brokers.

In view of the various modes of payments through electronic means available today,it is directed that Stock Brokers shall not accept cash from their clients either directly orby way of cash deposit to the bank account of stock broker.

All payments shall be received / made by the Stock Brokers from/to the clientsstrictly by account payee crossed cheques/ demand draft or by way of credit into thebank account through electronic fund transfer, or any other mode permitted by the ReserveBank of India. The stock brokers shall accept cheques drawn only by the clients andalso issue cheques in favour of the clients only, for their transactions. Stock Brokersshall not accept cash from their clients either directly or by way of cash deposit to thebank account of Stock Broker.

Answer 6A(iii)

Under the current regulatory framework, Sub-Brokers (‘SB’) need to seek registrationfrom SEBI under SEBI (Stock Broker and Sub-Broker) Regulations, 1992, and AuthorizedPersons (‘AP’) need to seek registration from the concerned Exchange. There is nodifference in the operative role of a Sub-Broker and that of an Authorized Person.Therefore, SEBI vide its circular no. SEBI/HO/MIRSD/DoP/CIR/P/2018/117 dated 03/08/2018 has discontinued with sub-broker as an intermediary to be registered with SEBI.In view of the same, the need for the category of the Sub-Broker as a market intermediarymay no longer be required. Therefore, it is decided that –

(a) No fresh registration shall be granted to any person as Sub-Broker. Any pendingapplications for registration as Sub-Broker under process, shall be returned tothe concerned Stock Exchanges for onward transmission to the applicant.

(b) The registered Sub-Brokers shall have time till March 31, 2019 in order to migrateto act as an Authorised Person (AP) and / or Trading Member (TM). The sub-Brokers, who do not choose to migrate into AP and / or TM, shall deemed to havesurrendered their registration with SEBI as Sub-Broker, w.e.f. March 31, 2019.

(c) Consequent upon migration/deemed surrender, the Certificate of Registrationgranted to the Sub-Broker by SEBI shall stand withdrawn.

Page 51: EXECUTIVE PROGRAMME (New Syllabus) · GUIDELINE ANSWERS EXECUTIVE PROGRAMME (New Syllabus) JUNE 2019 MODULE 2 ICSI House, 22, Institutional Area, Lodi Road, New Delhi 110 003 Phones:

49 EP–SLCM–June 2019

(d) The migration path for existing registered Sub-Broker, shall be under:

i. In case of a registered Sub-Broker who is already approved to act as AP inDerivatives Segment of the Exchanges, he shall be registered with theExchanges to continue activities of Sub-Broker as AP in Cash Segment.

ii. In case of a registered Sub-Broker who is not approved by Stock Exchangesto act as AP in Derivatives Segment, Exchanges shall register them as APin Cash Segment, to continue their operations without disruption.

iii. The existing Sub-Broker has an option to become a Trading Member, if theSub-Broker meets the eligibility criteria prescribed under Stock ExchangeBye-Laws and SEBI Regulations and by complying with these Regulations.

***

Page 52: EXECUTIVE PROGRAMME (New Syllabus) · GUIDELINE ANSWERS EXECUTIVE PROGRAMME (New Syllabus) JUNE 2019 MODULE 2 ICSI House, 22, Institutional Area, Lodi Road, New Delhi 110 003 Phones:

EP–EBCL–June 2019 50

ECONOMIC BUSINESS AND COMMERCIAL LAWS

Time allowed : 3 hours Maximum marks : 100

NOTE : Answer ALL Questions.

PART  IQuestion 1

(a) Reserve Bank of India is a banker of banks. Comment.

(b) How a person resident in India can hold, own, transfer or invest in any immovableproperty situated outside India. Comment.

(c) Explain under what circumstances the Central Government is empowered tosupersede any authority constituted under Special Economic Zones Act, 2005?What will be the consequences if such power is exercised by CentralGovernment ?

(d) What are the conditions for the Indian Company to allot sweat equity shares ofits holding company to its employees, who are resident outside India.

(5 marks each)Answer 1(a)

From the perusal of following functions of Reserve Bank of India (RBI), it is rightlysaid that Reserve Bank of India (RBI) is a Banker of Banks:

The Reserve Bank of India opens current accounts of banks with itself, enablingthese banks to maintain cash reserves as well as to carry out inter-bank transactionsthrough these accounts. Inter-bank accounts can also be settled by transfer of moneythrough electronic fund transfer system, such as, the Real Time Gross Settlement System(RTGS).

In addition, the Reserve Bank has also introduced the Centralised Funds ManagementSystem (CFMS) to facilitate centralised funds enquiry and transfer of funds acrossDeposit Accounts Department (DADs). This helps banks in their fund management asthey can access information on their balances maintained across different DADs from asingle location.

As Banker of Banks, the Reserve Bank provides short-term loans and advancesto selected banks, when necessary, to facilitate lending to specific sectors and forspecific purposes. These loans are provided against promissory notes and other collateralgiven by the banks.

The Reserve Bank also acts as the ‘lender of last resort’. It can come to the rescueof a bank that is solvent but faces temporary liquidity problems by supplying it withmuch needed liquidity when no one else is willing to extend credit to that bank. TheReserve Bank extends this facility to protect the interest of the depositors of the bankand to prevent possible failure of a bank, which in turn may also affect other banks andinstitutions and can have an adverse impact on financial stability and thus on the economy.

50

Page 53: EXECUTIVE PROGRAMME (New Syllabus) · GUIDELINE ANSWERS EXECUTIVE PROGRAMME (New Syllabus) JUNE 2019 MODULE 2 ICSI House, 22, Institutional Area, Lodi Road, New Delhi 110 003 Phones:

51 EP–EBCL–June 2019

Answer 1(b)

According to section 6(4) of the Foreign Exchange Management Act, 1999 read withForeign Exchange Management (Acquisition and Transfer of immovable property outsideIndia) Regulations, 2015, a person resident in India can hold, own, transfer or invest inany immovable property situated outside India if such property was acquired, held orowned by him/ her when he/ she was resident outside India or inherited from a personresident outside India.

A person resident in India can acquire immovable property outside India by way ofgift or inheritance from a person resident in India who has acquired such property inaccordance with the foreign exchange provisions in force at the time of such acquisition.

A person resident in India can purchase immovable property outside India out offoreign exchange held in his/ her Resident Foreign Currency (RFC) account.

A person resident in India can acquire immovable property outside India jointly witha relative who is a person resident outside India, provided there is no outflow of fundsfrom India.

Answer 1(c)

Section 40 of the Special Economic Zones Act, 2005 empowers the CentralGovernment to supersede an Authority for a maximum period of six months if at anytime, it is of the opinion that an Authority is unable to perform, or has persistently madedefault in the performance of the duty imposed on it or has exceeded or abused itspowers, or has wilfully or without sufficient cause, failed to comply with any directionissued by it. However, before issuing a notification superseding an authority, the CentralGovernment is required to give reasonable time to that Authority to make representationagainst the proposed suppression and consider the representations, if any, of theAuthority.

Section 40(2) of the Special Economic Zones Act, 2005 dealing with the consequencesof publication of the notification superseding the Authority provides that

(a) the Chairperson and other Members of the Authority shall, notwithstanding thattheir term of office has not expired as from the date of supersession, vacatetheir offices as such;

(b) all the powers, functions and duties which may, by or under the provisions ofthe Act, be exercised or discharged by or on behalf of the Authority shall, duringthe period of supersession, be exercised and performed by such person orpersons as the Central Government may direct;

(c) all property vested in the Authority shall, during the period of supersession, vestin the Central Government.

Answer 1(d)

Conditions for the Indian company to issue/allot sweat equity shares to its employees/directors or employees/directors of its holding company who are resident outside India,are as under:

(a) The scheme has been drawn either in terms of regulations issued under the

Page 54: EXECUTIVE PROGRAMME (New Syllabus) · GUIDELINE ANSWERS EXECUTIVE PROGRAMME (New Syllabus) JUNE 2019 MODULE 2 ICSI House, 22, Institutional Area, Lodi Road, New Delhi 110 003 Phones:

EP–EBCL–June 2019 52

Securities Exchange Board of India Act, 1992 or the Companies (Share Capitaland Debentures) Rules, 2014 notified by the Central Government under theCompanies Act 2013, as the case may be.

(b) The “sweat equity shares” issued to non-resident employees/directors underthe applicable rules/regulations are in compliance with the sectoral cap applicableto the said company.

(c) Issue of “sweat equity shares” by a company where foreign investment is underthe approval route shall require prior approval of Government of India.

(d) Issue of “sweat equity shares” under the applicable rules/regulations to anemployee/director who is a citizen of Bangladesh/Pakistan shall require priorapproval of the Government of India.

(e) The issuing company shall furnish to the Regional Office concerned of theReserve Bank of India under whose jurisdiction the registered office of thecompany operates, within 30 days from the date of issue of employees’ stockoption or sweat equity shares, a return as per the Form-ESOP.

Attempt all parts of either Q. No. 2 or Q. No. 2A

Question 2

(a) Explain the permissible capital account transactions by an individual under theliberalised remittance scheme. (4 marks)

(b) Explain the objectives of ‘Foreign Trade Policy’ under the Foreign Trade Policyfor 2015-2020. (4 marks)

(c) What do you understand by ‘‘Offshore Banking Unit’’ under Special EconomicZone Act, 2005 ? (4 marks)

(d) Whether conversion of ‘External Commercial Borrowing’ into equity is permissible?Comment. (4 marks)

(e) Discuss the objectives of enacting the Foreign Exchange Management Act,1999. Explain in brief the scheme of Foreign Exchange Management Act, 1999.

(4 marks)

OR (Alternate question to Q. No. 2)

Question 2A

(i) State the sources within which the Indian mutual funds registered with SEBI arepermitted to invest in overseas direct investment.

(ii) What are the reporting requirements for non-banking financial companies ?

(iii) Under what conditions ‘Foreign Direct Investment’ in limited liability partnershipis permitted.

(iv) How the ‘‘Special Economic Zone Authority’’ is constituted under SpecialEconomic Zone Act, 2005 ? Which are the defects or irregularities for which anyact or proceedings of an authority can not be invalidated ?

Page 55: EXECUTIVE PROGRAMME (New Syllabus) · GUIDELINE ANSWERS EXECUTIVE PROGRAMME (New Syllabus) JUNE 2019 MODULE 2 ICSI House, 22, Institutional Area, Lodi Road, New Delhi 110 003 Phones:

53 EP–EBCL–June 2019

(v) ‘Non-Banking Financial Companies’ are akin to that of bank, but they differ frombanks in certain cases. Explain. (4 marks each)

Answer 2(a)

The permissible capital account transactions by an individual under LiberalisedRemittance Scheme are:

(i) Opening of foreign currency account abroad with a bank.

(ii) Purchase of property abroad.

(iii) Making investments abroad- acquisition and holding shares of both listed andunlisted overseas company or debt instruments.

(iv) Acquisition of qualification shares of an overseas company for holding the postof Director, acquisition of shares of a foreign company towards professionalservices rendered or in lieu of Director’s remuneration.

(v) Investments in units of Mutual Funds, Venture Capital Funds, Unrated debtsecurities, promissory notes

(vi) Setting up wholly owned subsidiaries and Joint Ventures outside India for bonafidebusiness subject to the stipulated terms and conditions.

(vii) Extending loans including loans in Indian rupees to Non-Resident Indians (NRIs)who are relatives as defined in the Companies Act.

Answer 2(b)

India’s Foreign Trade Policy (FTP) has, conventionally, been formulated for fiveyears at a time and reviewed annually. The focus of the FTP has been to provide aframework of rules and procedures for exports and imports and a set of incentives forpromoting exports. The FTP for 2015-20 seeks to achieve the following:-

(i) To provide a stable and sustainable policy environment for foreign trade inmerchandise and services;

(ii) To link rules, procedures and incentives for exports and imports with otherinitiatives such as "Make in India", Digital India and Skill India to create an‘Export Promotion Mission’ for India;

(iii) To promote the diversification of India’s export by helping various sectors of theIndian economy to gain global competitiveness with a view to promote exports;

(iv) To create an architecture for India’s global trade engagement with a view toexpanding its markets and better integrating with major regions, thereby increasingthe demand for India’s product and contributing to the government’s flagship"Make in India" initiative;

(v) To provide a mechanism for regular appraisal in order to rationalise imports andreduce the trade imbalance.

Answer 2(c)

According to Section 2(u) of the Special Economic Zones Act, 2005, “OffshoreBanking Unit” means a branch of a bank located in a Special Economic Zone and which

Page 56: EXECUTIVE PROGRAMME (New Syllabus) · GUIDELINE ANSWERS EXECUTIVE PROGRAMME (New Syllabus) JUNE 2019 MODULE 2 ICSI House, 22, Institutional Area, Lodi Road, New Delhi 110 003 Phones:

EP–EBCL–June 2019 54

has obtained the permission under Section 23 (1)(a) of the Banking Regulation Act,1949.

An application for setting up and operation of an Offshore Banking Unit in a SpecialEconomic Zone may be made to the Reserve Bank, in the prescribed form and manner.The Reserve Bank may, by notification, specify the terms and conditions subject towhich an Offshore Banking Unit may be set up and operated in the Special EconomicZone.

Answer 2(d)

Indian companies have been granted general permission for conversion of ExternalCommercial Borrowings (ECB) (excluding those deemed as ECB) in convertible foreigncurrency into equity shares/fully compulsorily and mandatorily convertible preferenceshares, subject to the following conditions and reporting requirements:

(a) The activity of the company is covered under the Automatic Route for FDI orthe company has obtained Government approval for foreign equity in the company;

(b) The foreign equity after conversion of ECB into equity is within the sectoral cap,if any;

(c) Pricing of shares is as per the issue price of shares;

(d) Compliance with the requirements prescribed under any other statute andregulation in force; and

(e) The conversion facility is available for ECBs availed under the Automatic orGovernment Route and is applicable to ECBs, due for payment or not, as wellas secured/unsecured loans availed from non-resident collaborators.

Answer 2(e)

Foreign Exchange Management Act, 1999 enacted to consolidate and amend thelaw relating to foreign exchange with the objective of facilitating external trade andpayments and for promoting the orderly development and maintenance of foreign exchangemarket in India. In fact it is the central legislation that deals with inbound investmentsinto India and outbound investments from India and deal with trade and business betweenIndia and the other countries.

Foreign Exchange Management Act, 1999 makes provisions for dealings in foreignexchange. Broadly, all Current Account Transactions are free. However, CentralGovernment can impose reasonable restrictions by issuing Rules. Capital AccountTransactions are permitted to the extent specified by Reserve Bank of India (RBI) byissuing Regulations. Foreign Exchange Management Act, 1999 envisages that RBI shallhave a controlling role in management of foreign exchange. Since RBI cannot directlyhandle foreign exchange transactions, it authorizes “Authorised Persons” to deal in foreignexchange.

Foreign Exchange Management Act, 1999 also makes provisions for enforcement,penalties, adjudication and appeal also contains only basic legal framework. The practicalaspects are covered in Rules made by Central Government and Regulations made byRBI.

Page 57: EXECUTIVE PROGRAMME (New Syllabus) · GUIDELINE ANSWERS EXECUTIVE PROGRAMME (New Syllabus) JUNE 2019 MODULE 2 ICSI House, 22, Institutional Area, Lodi Road, New Delhi 110 003 Phones:

55 EP–EBCL–June 2019

Answer 2A(i)

Indian Mutual Funds registered with SEBI are permitted to invest in oversees directinvestment in the following:

a. American Depositary Receipts (ADRs) / Global Depositary Receipts (GDRs) ofthe Indian and foreign companies;

b. Equity of overseas companies listed on recognized overseas stock exchanges;initial and follow on public offerings for listing at recognized overseas stockexchanges;

c. Foreign debt securities- short term as well as long term with rating not belowinvestment grade - in the countries with fully convertible currencies;

d. Money market investments not below investment grade; repos where the counterparty is not below investment grade;

e. Government securities where countries are not rated below investment grade;

f. Derivatives traded on recognized stock exchanges overseas only for hedgingand portfolio balancing with underlying as securities;

g. Short term deposits with banks overseas where the issuer is rated not belowinvestment grade; and

h. Units / securities issued by overseas Mutual Funds or Unit Trusts registeredwith overseas regulators.

Answer 2A(ii)

The reporting requirements for Non-Banking Financial Companies are:-

(1) The Non-Banking Financial Company may, from time to time, prescribe return/s to be submitted by NBFC-P2P, as it deems fit.

(2) The following quarterly statements shall be submitted to the Regional Officewithin 15 days after the quarter to which these relate

— A statement, showing the number and amount in respect of loans;

• disbursed during the quarter;

• closed during the quarter; and

• outstanding at the beginning and at the end of the quarter, including thenumber of lenders and borrowers outstanding as at the end of the quarter

— The amount of funds held in the Escrow Account, bifurcated into fundsreceived from lenders and funds received from borrowers, with credit anddebit summations for the quarter.

— Number of complaints outstanding at the beginning and at the end of quarter,and disposed of during the quarter, bifurcated as received from lenders andborrowers.

— The Leverage Ratio, with details of its numerator and denominator.

Page 58: EXECUTIVE PROGRAMME (New Syllabus) · GUIDELINE ANSWERS EXECUTIVE PROGRAMME (New Syllabus) JUNE 2019 MODULE 2 ICSI House, 22, Institutional Area, Lodi Road, New Delhi 110 003 Phones:

EP–EBCL–June 2019 56

Answer 2A(iii)

Foreign Direct Investment (FDI) in Limited Liability Partnership (LLPs) is permittedsubject to the following conditions:

(i) FDI is permitted under the automatic route in Limited Liability Partnership (LLPs)operating in sectors/activities where 100% FDI is allowed through the automaticroute and there are no FDI linked performance conditions.

(ii) An Indian company or an LLP having foreign investment, is also permitted tomake downstream investment in another company or LLP in sectors in which100% FDI is allowed under the automatic route and there are no FDI-linkedperformance conditions.

(iii) FDI in LLP is subject to the compliance of the conditions of Limited LiabilityPartnership Act, 2008.

Answer 2A(iv)

Section 31 of Special Economic Zone Act,2005 empowers the Central Governmentto constitute by notification in the Official Gazette, Special Economic Zone Authority’for every Special Economic Zone to exercise powers conferred on and discharge thefunctions assigned to it.

Section 31(9) of Special Economic Zone Act, 2005 stipulates that no act orproceedings of an authority shall be invalidated merely by reason of:

(i) any vacancy in or any defect;

(ii) any defect in the appointment of a person as its member; or

(iii) any irregularity in the procedure of the authority not affecting the merits of thecase.

Answer 2A(v)

A Non-Banking Financial Company (NBFC) is a company registered under theCompanies Act, engaged in the business of loans and advances, acquisition of shares/stocks/bonds/debentures/securities issued by Government or local authority or othermarketable securities of a like nature, leasing, hire-purchase, insurance business, chitbusiness but does not include any institution whose principal business is that of agricultureactivity, industrial activity, purchase or sale of any goods (other than securities) orproviding any services and sale/purchase/construction of immovable property.

NBFCs lend and make investments and hence their activities are akin to that ofbanks; however there are few differences such as:

(i) NBFC cannot accept demand deposits.

(ii) NBFC’s do not form part of the payment and settlement system and cannotissue cheques drawn on itself.

(iii) Deposit insurance facility of Deposit Insurance and Credit Guarantee Corporationis not available to depositors of NBFC’s unlike in the case of banks.

Page 59: EXECUTIVE PROGRAMME (New Syllabus) · GUIDELINE ANSWERS EXECUTIVE PROGRAMME (New Syllabus) JUNE 2019 MODULE 2 ICSI House, 22, Institutional Area, Lodi Road, New Delhi 110 003 Phones:

57 EP–EBCL–June 2019

PART–II

Question 3

(a) How the competition commission will determine whether an agreement hasappreciable adverse effect on competition ?

(b) Whether the jurisdiction of the competition commission of India extends to acts/agreements taking place outside India, which affects competition in India ?Explain.

(c) What is the purpose of competition policy of India and Competition Act, 2002 ?

(d) What is meant by dominant position under the Competition Act, 2002 ?

(e) Discuss the consequences of making false statement by a person being a partyto combination under Competition Act, 2002. (3 marks each)

Answer 3(a)

Section 19(3) of the Competition Act,2002 provides that while determining whetheran agreement has appreciable adverse effect on competition, the Commission shallgive due regard to all or any of the following factors, namely:

(a) creation of barriers to new entrants in the market;

(b) driving existing competitors out of the market;

(c) foreclosure of competition by hindering entry into the market;

(d) accrual of benefits to consumers;

(e) improvements in production or distribution of goods or provision of services;

(f) promotion of technical, scientific and economic development by means ofproduction or distribution of goods or provision of services.

Answer 3(b)

Section 32 of the Competition Act, 2002 deals with acts taking place outside Indiabut having an effect on competition in India. Section 32 of the Competition Act, 2002extends the jurisdiction of Competition Commission of India to inquire and pass ordersin accordance with the provisions of the Act into an agreement or dominant position orcombination, which is likely to have, an appreciable adverse effect on competition inrelevant market in India, notwithstanding that:

(a) an agreement referred to in Section 3 has been entered into outside India; or

(b) any party to such agreement is outside India; or

(c) any enterprise abusing the dominant position is outside India; or

(d) a combination has taken place outside India; or

(e) any party to combination is outside India; or

(f) any other matter or practice or action arising out of such agreement or dominantposition or combination is outside India.

Page 60: EXECUTIVE PROGRAMME (New Syllabus) · GUIDELINE ANSWERS EXECUTIVE PROGRAMME (New Syllabus) JUNE 2019 MODULE 2 ICSI House, 22, Institutional Area, Lodi Road, New Delhi 110 003 Phones:

EP–EBCL–June 2019 58

The above clearly demonstrates that acts taking place outside India but having aneffect on competition in India will be subject to the jurisdiction of Commission. TheCompetition Commission of India will have jurisdiction even if both the parties to anagreement are outside India but only if the agreement, dominant position or combinationentered into by them has an appreciable adverse effect on competition in the relevantmarket of India.

Answer 3(c)

The basic purpose of Competition Policy and Competition Act, 2002 is to preserveand promote competition as a means of ensuring efficient allocation of resources in aneconomy. Competition policy typically has two elements:

(i) One is a set of policies that enhance competition in local and national markets.

(ii) The second element is legislation designed to prevent anti-competitive businesspractices with minimal Government intervention, i.e., a competition law.

The Competition Act, 2002 seeks to provide, keeping in view the economicdevelopment of the country, for the establishment of Competition Commission to preventpractices having adverse effect on competition, to promote and sustain competition inmarkets, to protect the interests of consumers and to ensure freedom of trade carried onby other participants in markets in India.

Answer 3(d)

According to Section 4 of the Competition Act, 2002 dominant position means aposition of strength, enjoyed by an enterprise or group, in the relevant market, in India,which enables it to–

(i) operate independently of competitive forces prevailing in the relevant market; or

(ii) affect its competitors or consumers or the relevant market in its favour”.

Answer 3(e)

Section 44 of the Competition Act, 2002 provides that if any person, being a party toa combination, makes a statement which is false in any material particular, or knowingit to be false; or omits to state any material particular knowing it to be material, suchperson shall be liable to a penalty which shall not be less than rupees fifty lakhs butwhich may extend to rupees one crore, as may be determined by the Commission.

Question 4

(a) The Competition Act, 2002 is an improvement on the MRTP Act, 1969. Criticallyanalyse this statement.

(b) Explain what is meant by Bid-rigging, tie-in agreement, Exclusive supplyagreement, and Refusal to deal. (5 marks each)

Answer 4(a)

It is true that the Competition Act, 2002 is an improvement on the MRTP Act, 1969.

Page 61: EXECUTIVE PROGRAMME (New Syllabus) · GUIDELINE ANSWERS EXECUTIVE PROGRAMME (New Syllabus) JUNE 2019 MODULE 2 ICSI House, 22, Institutional Area, Lodi Road, New Delhi 110 003 Phones:

59 EP–EBCL–June 2019

Following differences in the MRTP Act, 1969 and the Competition Act, 2002 support thisstatement:

(i) Difference in goals as stated in Preamble of the Acts

• The MRTP Act,1969 aimed at prevention of concentration of economicpower and prohibition of monopolistic, restrictive and unfair trade practiceswhile objective of the Competition Act,2002 is to promote and sustaincompetition, protect the interest of consumers and ensure freedom of tradecarried on by other participants, in markets in India.

• The broad objective of MRTP Act, 1969 was to control monopolies whilethe objective of the Competition Act 2002 is to promote competition.

(ii) Difference in provisions w.r.t. dominance

• According to MRTP Act, 1969, dominance per se is bad while the provisionsof Competition Act, 2002 states that it’s not dominance which is prohibitedbut it is abuse of dominance which is per se bad.

(iii) Regulation of Merger & Acquisitions (Combination)

• MRTP Act, 1969 did not provide for regulation of merger& acquisition whilethe Competition Act, 2002 provides for regulation of merger & acquisitionwhich is absolutely important in the new regime of liberalization.

(iv) Provision for penalties

• MRTP Act, 1969 did not provide for penalties specifically while theCompetition Act, 2002 specifically provides for imposition of penalties forviolations under the Act.

(v) Advocacy provisions

• There were no advocacy provisions for Monopolistic and Restrictive TradePractices Commission while the Competition Act, 2002 mandatesCompetition Advocacy provisions for Competition Commission of India.

(vi) Extra-territorial jurisdiction of Commission• Monopolistic and Restrictive Trade Practices Commission did not have extra-

territorial jurisdiction beyond India in appropriate cases while CompetitionCommission of India is vested with extra-territorial jurisdiction in appropriatecases having adverse effect on competition in India.

Answer 4(b)

Bid Rigging : According to Section 3 of the Competition Act, 2002 “bid rigging”means any agreement, between enterprises or persons engaged in identical or similarproduction or trading of goods or provision of services, which has the effect of eliminatingor reducing competition for bids or adversely affecting or manipulating the process forbidding

Tie-in agreement : Section 3 of the Competition Act, 2002, defines the term “tie-inagreement” to include any agreement requiring a purchaser of goods, as a condition of

Page 62: EXECUTIVE PROGRAMME (New Syllabus) · GUIDELINE ANSWERS EXECUTIVE PROGRAMME (New Syllabus) JUNE 2019 MODULE 2 ICSI House, 22, Institutional Area, Lodi Road, New Delhi 110 003 Phones:

EP–EBCL–June 2019 60

such purchase, to purchase some other goods. A good example of tie-in agreement iswhere a gas distributor requires a consumer to buy a gas stove as a pre-condition toobtain connection of domestic cooking gas.

Exclusive supply agreement : Section 3 of the Competition Act, 2002, defines theterm Exclusive supply agreement to include any agreement restricting in any mannerfrom acquiring or otherwise dealing in any goods other than those of the seller or anyother person. Thus, where a manufacturer asks a dealer not to deal in similar productsof its competitor directly or indirectly and discontinues the supply on the ground thatdealer also deals in product of suppliers’ competitor’s goods is an illustration of exclusivedealing agreement.

Refusal to deal : According to Section 3 of the Competition Act, 2002, the term“Refusal to deal” includes any agreement, which restricts, or is likely to restrict, by anymethod the persons or classes of persons to whom goods are sold or from whom goodsare bought. For e.g. an agreement which provides that the franchisees will not deal inproducts or goods of similar nature for a period of three years from the date of determinationof agreement within a radius of five kms from showroom amounts to exclusive dealingagreement.

PART–III

Question 5

(a) Ram employed in Mumbai promised to pay ̀ 8,000 per month to his wife Sunita.She was living in Delhi. On receiving information that she has become unfaithfulto him, Ram stopped the payment of ̀ 8,000 to Sunita. Sunita approaches to filea case against Ram. Advise her with reference to the Indian Contract Act, 1872.

(4 marks)

(b) Rajni got herself operated in the Devashri Hospital for removal of her uterus, asa cyst was found to have developed near one of her ovaries. The surgeon, whoperformed the operation, left abdominal pack in the abdominal. This caused lotof pain, suffering and uneasiness to her. The abdominal pack was subsequentlyremoved by another surgical operation. It was alleged that due to negligence ofthe surgeon, the patient suffered all the consequences, therefore it should betreated as negligence in the treatment. But the Hospital authorities contendedthat the patient’s problem was removed by the second operation, hence it is notdeficiency. Rajni sought the relief under Consumer Protection Act, 1986 statingthat this negligence may be treated as deficiency and compensation may bepaid to her. Decide whether Ranji will succeed or not ? Why ? (4 marks)

(c) A draws and B accepts the bill payable to C or order. C endorses the bill to Dand D to E, who is ‘holder in due course’. Decide from whom E can recover theamount under the Negotiable Instrument Act, 1881 ? (4 marks)

(d) Sachin made an unconditional gift of property to Amit but continued in possessionof gifted property. Sachin revoked the gift deed transferred it to Naresh. Amitwants to recover possession from Naresh. Discuss it in the light of provisions ofTransfer of Property Act, 1882 whether Naresh can withhold the gifted property?

(4 marks)

Page 63: EXECUTIVE PROGRAMME (New Syllabus) · GUIDELINE ANSWERS EXECUTIVE PROGRAMME (New Syllabus) JUNE 2019 MODULE 2 ICSI House, 22, Institutional Area, Lodi Road, New Delhi 110 003 Phones:

61 EP–EBCL–June 2019

(e) A minor fraudulently overstated his age and purchased a motor car after executinga promissory note in favour of the owner of the motor car for its price. The carowner compelled the minor to pay the amount of the promissory note. Whetherthe car owner will succeed ? Examine it with reference to Indian Contract Act,1872 and Specific Relief Act, 1963. (4 marks)

Answer 5(a)

According to section 2(h) of the Indian Contract Act, 1872, Contract is an agreementenforceable by law i.e. for an agreement to become a contract, there must be intentionto create legal relationship.

In the present case, intention to create legal relationship is absent as this is adomestic arrangement between husband and wife. According to the Indian Contract Act,1872, it does not make any difference whether the agreement is oral or in writing. Sobeing domestic arrangement between husband and wife, it is presumed that intention tocreate legal relationship is absent in the present case and therefore, this agreement isnot enforceable. Therefore, Sunita would not succeed in the present case. (Ref. Balfourv. Balfour)

Answer 5(b)

According to section 2(1)(g) of the Consumer Protection Act, 1986, “Deficiency”means any fault, imperfection, shortcoming or inadequacy in the quality, nature andmanner of performance which is required to be maintained by or under any law for thetime being in force or has been undertaken to be performed by a person in pursuance ofa contract or otherwise in relation to any service.

The principles, laid down by Supreme Court in the case of Kusum Sharma & Othersversus Batra Hospital & Medical Research Centre & Others Supreme Court (CP), shallbe kept in view while deciding the cases of medical negligence. As long as the doctorshave performed their duties and exercised an ordinary degree of professional skill andcompetence, they cannot be held guilty of medical negligence.

In the present case the doctor is liable for medical negligence as by leaving abdominalpack in the abdomen, his conduct fell below that of the standards of a reasonablycompetent practitioner in his field. The doctor has breached all the above stated principles.Therefore, he is liable for medical negligence.

Rajni will succeed in this case and will be entitled for compensation.

Answer 5(c)

According to the Negotiable Instrument Act, 1881, every prior party to a negotiableinstrument is liable thereon to a holder in due course until the instrument is duly satisfied.Prior parties may include the maker or drawer, the acceptor and all the interveningendorsers to a negotiable instrument. The liability of the prior parties to a holder in duecourse is joint and several. The holder in due course may hold any or all prior partiesliable for the amount of the dishonoured instrument.

Therefore, in present case, “E” can recover the amount payable on bill from:

(i) “A” being drawer of bill having secondary and conditional liability arising in caseof dishonor of bill.

Page 64: EXECUTIVE PROGRAMME (New Syllabus) · GUIDELINE ANSWERS EXECUTIVE PROGRAMME (New Syllabus) JUNE 2019 MODULE 2 ICSI House, 22, Institutional Area, Lodi Road, New Delhi 110 003 Phones:

EP–EBCL–June 2019 62

(ii) “B” being drawee and acceptor of bill having primary and absolute liability to paybill.

(iii) “C” and “D” being endorsers and prior parties to the bill.

Answer 5(d)

According to Section 122 of the Transfer of Property Act, once a gift is accepted bythe donee/his legal representatives during life time of donor, the donor cannot revoke it.

One of the essential feature of gift is that it cannot be revoked at the will andpleasure of the grantor. A gift which comes into existence on the fulfilment of a condition,that is to say, a gift which is subject to a condition precedent is also valid.

But in this case, as gift is unconditional, therefore, after its acceptance, Sachincannot revoke gift deed.

Applying section 41 of the Transfer of Property Act dealing with transfer by ostensibleowner, in present case, Sachin (donor) is not ostensible owner as he is not holding theproperty with the consent of Amit (real owner). Therefore, Naresh (transferee), cannot,invoke the protection of Section 41. Amit would succeed in the case and Naresh cannotwithhold the gifted property.

Answer 5(e)

According to the Indian Contract Act, 1872, a minor has no capacity to contract andminor's contract is absolutely void. Law of estoppel is not applicable to a minor and hecan always plead his minority and is not estopped from doing so even where he hadproduced a loan or entered into some other contract by falsely representing that he wasof full age, when in reality he was a minor.

But where the loan was obtained by fraudulent representation by the minor or someproperty was sold by him and the transactions are set aside as being void, the Court onequitable grounds may direct the minor to restore the property to the other party.

According to Section 33 of the Specific Relief Act, 1963 the Court may, if the minorhas received any benefit under the agreement from the other party require him to restore,so far as may be such benefit to the other party, to the extent to which he or his estatehas been benefited thereby.

In present case, car owner will not succeed in getting the amount of promissory notebut he can get back his motor car.

Attempt all parts of either Q. No. 6 or Q. No. 6A

Question 6

Distinguish between the following :

(a) Contract of service and Contract for service

(b) Specific performance and Injunction

(c) Cheque and Bill of Exchange

(d) Mortgage and Charge(e) Conditions and Warranties. (3 marks each)

Page 65: EXECUTIVE PROGRAMME (New Syllabus) · GUIDELINE ANSWERS EXECUTIVE PROGRAMME (New Syllabus) JUNE 2019 MODULE 2 ICSI House, 22, Institutional Area, Lodi Road, New Delhi 110 003 Phones:

63 EP–EBCL–June 2019

OR (Alternate question to Q. No. 6)

Question 6A

(i) Explain the salient features of the Benami Transactions (Prohibition) Act, 1998.(3 marks)

(ii) What are the effects of Money laundering on the economic development of acountry ? (3 marks)

(iii) Explain the functions of ‘Real Estate Regulatory Authority’ under the Read Estate(Regulation and Development) Act, 2016. (3 marks)

(iv) Whether a minor may be admitted in the business of partnership firm under theIndian Partnership Act, 1932 ? Whether the minor will have any right in parnershipfirm ? Explain. (3 marks)

(v) Against whom specific performance of a contract may be enforced under theSpecific Relief Act, 1963 ? Explain. (3 marks)

Answer 6(a)

A contract for service implies a contract whereby one party undertakes to renderservices e.g. professional or technical services to or for another in the performance ofwhich he is not subject to detailed direction and control but exercises professional ortechnical skill and uses his own knowledge and discretion.

A contract of service on the other hand implies relationship of master and servantand involves an obligation to obey orders in the work to be performed and as to its modeand manner of performance.”

Answer 6(b)

(a) Specific performance means the actual carrying out by the parties of their contract,and in proper cases the Court will insist upon the parties carrying out thisagreement. An injunction, is an order of a Court restraining a person from doinga particular act.

(b) Injunction is a mode of securing the specific performance of a negative term ofthe contract, (i.e., where he is doing something which he promises not to do),the Court may in its discretion issue an order to the defendant restraining himfrom doing what he promised not to do.

(c) Injunction may be prohibitory or mandatory. In prohibitory, the Court restrainsthe commission of a wrongful act whereas in mandatory, it restrains continuanceof a wrongful commission. Specific performance involves positive approach bythe Court to direct the parties to do what they have promised under the contract.

Answer 6(c)

(a) A cheque is a bill of exchange and always drawn on a banker, while a bill ofexchange may be drawn on any one, including banker.

(b) A cheque can only be drawn payable on demand, a bill of exchange may be drawnpayable on demand, or on the expiry of a specified period after sight or date.

Page 66: EXECUTIVE PROGRAMME (New Syllabus) · GUIDELINE ANSWERS EXECUTIVE PROGRAMME (New Syllabus) JUNE 2019 MODULE 2 ICSI House, 22, Institutional Area, Lodi Road, New Delhi 110 003 Phones:

EP–EBCL–June 2019 64

(c) A bill of exchange payable after sight must be accepted before payment can bedemanded, a cheque does not require acceptance and is intended for immediatepayment.

(d) A grace of three days is allowed in the case of time bills of exchange, while nograce is given in the case of a cheque, for payment.

(e) The drawer of a bill of exchange is discharged, if it is not presented for payment,but the drawer of a cheque is discharged only if he suffers any damage by delayin presentment for payment.

(f) Notice of the dishonour of a bill of exchange is necessary, but not in the case ofa cheque.

(g) The cheque being a revocable mandate, the authority may be revoked bycountermanding payment, and is determined by notice of the customer’s deathor insolvency. This is not so in the case of bill of exchange.

(h) A cheque may be crossed, but not a bill of exchange.

Answer 6(d)

(a) A mortgage is transfer of an interest in the property made by the mortgagor asa security for the loan, while the charge is not the transfer of any interest in theproperty though it is security for the payment of an amount.

(b) A charge may be created by act of parties or by operation of law. A mortgagecan only be created by act of parties.

(c) A mortgage deed must be registered and attested by two witnesses, while acharge need not be made in writing, and if reduced to writing, it need not beattested or registered.

(d) In certain types of mortgage (viz., mortgage by conditional sale and anomalousmortgage) the mortgagor can foreclose the mortgaged property but in charge,the charge-holder cannot foreclose though he can get the property sold as in asimple mortgage.

(e) From the very nature of it, a charge as a general rule, cannot be enforcedagainst a transferee for consideration without notice. But in a mortgage, thetransferee of mortgaged property from the mortgagor, can only acquire theremaining interest of the mortgagor, and is therefore, only bound by the mortgage.

(f) In a charge created by act of parties the specification of the particular fund orproperty negatives a personal liability and the remedy of the charge-holder isagainst the property only. In a mortgage, there can be security as well as personalliability. In fact, the absence of a personal liability is the principal test thatdistinguishes a charge from a simple mortgage.

Answer 6(e)

According to section 12 of the Sales of Goods Act, 1930, the difference between acondition and a warranty are:-

(a) If the stipulation forms the very basis of the contract or is essential to the mainpurpose of the contract, it is a condition. If the stipulation is collateral to themain purpose of the contract, i.e., is a subsidiary promise, it is a warranty.

Page 67: EXECUTIVE PROGRAMME (New Syllabus) · GUIDELINE ANSWERS EXECUTIVE PROGRAMME (New Syllabus) JUNE 2019 MODULE 2 ICSI House, 22, Institutional Area, Lodi Road, New Delhi 110 003 Phones:

65 EP–EBCL–June 2019

(b) The breach of the condition gives the aggrieved party a right to treat the contractas repudiated. The effect of a breach of a warranty is that the aggrieved partycannot repudiate the contract but can only claim damages.

(c) If the seller fails to fulfil a condition, the buyer may treat the contract asrepudiated, refuse the goods and, if he has already paid for them, recover theprice. He can also claim damages for the breach of contract. If the seller doesnot fulfil a warranty, the buyer must accept the goods and claim damages forbreach of warranty.

Answer 6A(i)

The Salient Features of the Benami Transactions (Prohibition) Act, 1988 are asunder:

(i) It defines a benami transaction and benami property and also provides forexclusions and transactions which shall not be construed benami.

(ii) It provides the consequences of entering into a prohibited benami transactions.

(iii) It lays down the procedure for determination and related penal consequences inthe case of a prohibited benami transaction.

(iv) It also provides that the powers of civil court shall be available to authoritiesunder the said Act.

(v) It empowers Central Government to make rules for the implementation of theprovisions of the Act

(vi) It enables the Central Government in consultation with the Chief Justice of theHigh Court to designate one or more Courts of Session as Special Court orSpecial Courts

(vii) It provides penalty for entering into benami transactions and for furnishing anyfalse documents in any proceeding under the Act

Answer 6A(ii)

The possible social, economic and political effects of money laundering, if leftunchecked or dealt with ineffectively, are serious. Through the process of moneylaundering, organised crime can infiltrate financial institutions, acquire control of largesectors of the economy through investment, or offer bribes to public officials and indeedgovernments. Thus, the economic and political influence of criminal organisations canweaken the social fabric, ethical standards and ultimately the democratic institutions ofsociety. There is a damping effect on foreign direct investment when a country’scommercial and financial sectors are perceived to be subject to the control and influenceof organised crime.

Answer 6A(iii)

The functions of “Real Estate Regulatory Authority” under the Real Estate (Regulationand Development) Act, 2016 are:-

(a) to register and regulate real estate projects and real estate agents registeredunder the Act;

Page 68: EXECUTIVE PROGRAMME (New Syllabus) · GUIDELINE ANSWERS EXECUTIVE PROGRAMME (New Syllabus) JUNE 2019 MODULE 2 ICSI House, 22, Institutional Area, Lodi Road, New Delhi 110 003 Phones:

EP–EBCL–June 2019 66

(b) to publish and maintain a website of records, for public viewing, of all real estateprojects for which registration has been given, with such details as may beprescribed, including information provided in the application for which registrationhas been granted;

(c) to maintain a database, on its website, for public viewing, and enter the namesand photographs of promoters as defaulters including the project details,registration for which has been revoked or have been penalised under the Act,with reasons therefor, for access to the general public;

(d) to maintain a database, on its website, for public viewing, and enter the namesand photographs of real estate agents who have applied and registered underthe Act, with such details as may be prescribed, including those whose registrationhas been rejected or revoked;

(e) to fix through regulations for each areas under its jurisdiction the standard feesto be levied on the allottees or the promoter or the real estate agent, as the casemay be;

(f) to ensure compliance of the obligations cast upon the promoters, the allotteesand the real estate agents under the Act and the Rules and Regulations madethereunder;

(g) to ensure compliance of its regulations or orders or directions made in exerciseof its powers under the Act;

(h) to perform such other functions as may be entrusted to the Authority by theappropriate Government as may be necessary to carry out the provisions of theAct.

Answer 6A(iv)

Drawing a joint inference from reading of Section 11 of the Indian Contract Act,1872, the decision of the Privy Council in Mohori Bibi v. Dharmo Das Ghose andSection 5 of the Indian Partnership Act, 1932, a minor being incompetent to contractcannot enter into agreement of partnership. The only provision that Section 30 of theIndian Partnership Act, 1932 makes is that with the “consent of all the partners for thetime being, a minor can be admitted into the benefits of partnership to which a minor isgoing to be admitted”. A partnership firm cannot be formed with only minors as partners.There must be at least two major partners before a minor is admitted into the benefits ofpartnership.

Rights of Minor as a partner:

(i) He is entitled to his agreed share and can inspect books of account of the firm[Section 30(2)].

(ii) He can bring a suit for account and his share when he intends to sever hisconnections with the firm, but not otherwise. [Section 30(4)]

Answer 6A(v)

According to Section 19 of Specific Relief Act, 1963, specific performance of acontract may be enforced against:

(a) either party thereto,

Page 69: EXECUTIVE PROGRAMME (New Syllabus) · GUIDELINE ANSWERS EXECUTIVE PROGRAMME (New Syllabus) JUNE 2019 MODULE 2 ICSI House, 22, Institutional Area, Lodi Road, New Delhi 110 003 Phones:

67 EP–EBCL–June 2019

(b) any person claiming under him, by a title arising subsequently to the contractexcept a transferee for value who has paid his money in good faith and withoutnotice of the original contract,

(c) any person claiming under a title which though prior to the contract, and knownto the plaintiff, might have been displaced by the defendant,

(d) when a limited liability partnership has entered into a contract and subsequentlybecomes amalgamated with another limited liability partnership, the new limitedliability partnership which arises out of the amalgamation

(e) when a company has entered into a contract and subsequently becomesamalgamated with another company, the new company which arises out of theamalgamation,

(f) when the promoters of a company have before its incorporation entered into acontract, for the purpose of the company and such contract is warranted by theterms of the incorporation of the company; provided that the company hasaccepted the contract and communicated such acceptance to the other party tothe contract.

***

Page 70: EXECUTIVE PROGRAMME (New Syllabus) · GUIDELINE ANSWERS EXECUTIVE PROGRAMME (New Syllabus) JUNE 2019 MODULE 2 ICSI House, 22, Institutional Area, Lodi Road, New Delhi 110 003 Phones:

EP–FSM–June 2019 68

FINANCIAL AND STRATEGIC MANAGEMENT

Time allowed : 3 hours Maximum marks : 100

NOTE : 1. Answer ALL Questions.2. Suitable assumptions, if considered necessary, may be made while answering

a question. However, such assumptions must be stated clearly.

3. Working notes should form part of the answer.

68

PART I

Question 1

Comment on the following :

(a) Investment, financing and dividend decisions are inter-related finance functions.

(b) Term loan is associated with some specific features.

(c) Legal, Contractual and Internal Constraints and Restrictions needs to beconsidered while framing dividend policy. (5 marks each)

Answer 1(a)

Financial Management is concerned with investment, financing and dividend decisionsin relation to objectives of the company. Investment ordinarily means utilisation of moneyfor profits or returns. Investment decisions are concerned with addition to capital assetsin view to increase the net worth of the firm. Financing decision is the next step infinancial management for executing the investment decision once taken. Financialdecisions encompass determination of the proportion of equity capital to debt to achievean optimal capital structure, and to balance the fixed and working capital requirementsin the financial structure of the company. The dividend decision is another major area offinancial management. The financial manager must decide whether the firm shoulddistribute whole profits/part profits or retain them.

Answer 1(b)

Features of Term Loan

1. Loan in any Currency : These loans are provided both in home or foreign currency.Home currency loans are offered normally for purchase of fixed assets such asland, building, plant and machineries, preliminary and preoperative expenses,technical know-how, working capital etc. On the other hand, foreign currencyloans are offered for import of certain plant or machinery, payment of foreignconsulting fee etc.

2. Secured Loan : Term loans come under secured category of loans. Two kindsof securities are there –primary and collateral.

3. Loan Installments : Repayment of loan is done in installments. These installmentscover both principal and interest. Normally, loan installments are decided by

Page 71: EXECUTIVE PROGRAMME (New Syllabus) · GUIDELINE ANSWERS EXECUTIVE PROGRAMME (New Syllabus) JUNE 2019 MODULE 2 ICSI House, 22, Institutional Area, Lodi Road, New Delhi 110 003 Phones:

69 EP-FSM–June 2019

banks based the borrower’s cash flow capacity. There may be installments paidmonthly, quarterly, biannually, or even annually.

4. Maturity : Normally a term loan is ranging between 5 to 10 years. Forecasting formore than 10 years in the current changing business environment is very difficult.

5. Loan Agreement : An agreement is drafted between the borrower and the bankregarding the terms and conditions of the loans which is signed by the borrowerand is preserved with the bank.

6. Loan Covenant : Covenants are a part of loan agreement. They are certainstatements in the agreement which states about certain do’s and dont’s for thecompany.

Answer 1(c)

Legal as well as contractual requirements have to be followed while framing thedividend policy. All provisions of Companies Act, SEBI Guidelines, Capital impairmentguidelines, net profit and insolvency etc. need to comply with while declaring dividend.

Contractual restrictions which are imposed by certain lenders of the firm also affectthe dividend policy of a firm as they impose certain conditions about the payment ofdividend particularly, during the period when the firm is experiencing liquidity or profitabilitycrisis. In addition, there may be certain internal constraints which affect the dividendpolicy of a firm; it includes growth prospects, financial requirements, availability of funds,earning stability and control etc.

Attempt all parts of either Q. No. 2 or Q. No. 2A

Question 2

(a) Equity shares of YMC Corporation has a beta (), of 1.4. The risk-free rate is9%, and the market return is 21%. Determine the risk premium on its equityshares. Further determine cost of equity using the suitable model. (3 marks)

(b) An investor is seeking an efficient portfolio with a correlation of 0.7 between theportfolio and the market and a standard deviation of 2.5%. The market standarddeviation is 1.4% and the market rate of return is 16%, a rate that is double thereturn on risk-free securities. What is the required rate of return being sought bythe investor ? (3 marks)

(c) A product requires 40 types of material every day for its manufacturing. A fixedcost of `50 per order is incurred for placing an order. The inventory carryingcost for each material is ̀ 0.02 per day. The lead period is 50 days. Assume 365days in a year. Calculate the following :

(i) Economic Order Quantity (EOQ)(ii) Re-order Level (ROL). (3 marks)

(d) Calculate the Accounting Rate of Return (ARR) and rank the machine accordingly:

Particulars Machine A Machine B

Initial Investment `87,500 `87,500

Page 72: EXECUTIVE PROGRAMME (New Syllabus) · GUIDELINE ANSWERS EXECUTIVE PROGRAMME (New Syllabus) JUNE 2019 MODULE 2 ICSI House, 22, Institutional Area, Lodi Road, New Delhi 110 003 Phones:

EP–FSM–June 2019 70

Cash flow after tax

Year 1 26,250 8,750Year 2 35,000 26,250Year 3 43,750 35,000Year 4 26,250 52,500Year 5 17,500 35,000

Machine has to be written off over a period of 5 years by straight line method.(3 marks)

(e) From the following information, calculate the expected rate of return of a portfolio:

Risk-free rate of return 10%Expected return on market portfolio 20%Standard deviation of an asset 3%Market standard deviation 2.8%Correlation coefficient of portfolio with market 0.70 (3 marks)

OR (Alternate question to Q. No. 2)

Question 2A

(i) A Private Ltd. Co. has assets worth `32,00,000 as on 31st March, 2018 thathas been financed as follows :

`

Equity Shares of `100 each 18,00,000

General Reserve 3,60,000

Debt 10,40,000

For the year ended on 31st March, 2018, the Company’s total profits beforeinterest and taxes were ̀ 6,23,000. The company pays 8% interest on borrowedcapital and the tax bracket is 40%. The market value of equity share as on 31stMarch, 2018 was ̀ 150 per share.

Calculate the weighted average cost of capital using market value as weights.(5 marks)

(ii) The total turnover of X Ltd. is `50 lakhs of which 72% is on credit. Debtors areallowed one month to clear off the dues. A factoring company is willing to advance80% of the bill raised on credit for a fee of 2% a month plus a commission of10% on the total amount of debts. X Ltd., as a result of this arrangement, islikely to save ̀ 48,000 annually in management cost and avoid bad debts @ 1%on the credit sales.

A bank has come forward to make an advance equal to 80% of the debts at anannual interest rate of 15%. However, its processing fees will be @ 2% on thedebts. Would you accept factoring or the offer from the bank ? (5 marks)

Page 73: EXECUTIVE PROGRAMME (New Syllabus) · GUIDELINE ANSWERS EXECUTIVE PROGRAMME (New Syllabus) JUNE 2019 MODULE 2 ICSI House, 22, Institutional Area, Lodi Road, New Delhi 110 003 Phones:

71 EP-FSM–June 2019

(iii) ABC. Ltd. has a risk free asset yielding 0.02. During this period expected returnof 0.09 and a standard deviation of 0.04 are based on past data of the marketportfolio. The risk premium 0.07 is considered to be constant throughout theperiod. A security has a coefficient of correlation with market portfolio of 0.75and standard deviation is 0.08. The expected standard deviation of marketportfolio is now 0.04. On the basis of the given data, find out :

(i) Market’s risk return trade-off; and (3 marks)

(ii) Security beta. (2 marks)

Answer 2(a)

Risk Premium = Market rate of Return (Rm) – Risk free rate of return (Rf)

= 21% - 9% = 12%

Cost of Equity (Ke) as per CAPM formula

Ke = Rf + ß (Rm-Rf) where,

Ke = Cost of Equity

Rf = Risk free rate of Return

ß = Risk Factor

Rm = Market Return

Hence, Cost of Equity (Ke) = 9% + (21%-9%) 1.4

= 25.8%

Answer 2(b)

Calculation of beta co-efficient

Standard deviation ( s) = 2.5%

Correlation co-efficient (rm)= 0.70

Market Standard Deviation ( m) = 1.4%

Beta Coefficient = (rm x s) ÷ m = (0.70 x 2.5) ÷ 1.4 = 1.25

Rm = 16 % Rf = 8 %

Required rate of return = Rf + ß (Rm-Rf)

= 8% +1.25 (16-8)

= 18 %

Answer 2(c)

(i) Annual Consumption (A) = 40 types of material x 365 days

= 14,600 types of material

Fixed Cost per order or ordering cost (O) = ̀ 50

Carrying Cost (C) = 0.02 x 365 days = `7.30

Page 74: EXECUTIVE PROGRAMME (New Syllabus) · GUIDELINE ANSWERS EXECUTIVE PROGRAMME (New Syllabus) JUNE 2019 MODULE 2 ICSI House, 22, Institutional Area, Lodi Road, New Delhi 110 003 Phones:

EP–FSM–June 2019 72

Economic Order Quantity (EOQ) = )C/AO2(

A = Annual Consumption = 14,600 types of material

O = Ordering Cost per order = ̀ 50

C = Carrying Cost = ̀ 7.30

EOQ = 50 x 600,14x2( / 7.30 = 447.21

(ii) Re-order level (ROL)= Maximum Usage per day x lead time

= 40 types of material per day x 50 days

= 2000 types of material

Answer 2(d)

Amount in `

Total Cash Flow after Tax (CFAT) 1,48,750 1,57,500

Less : Depreciation (machine has life of 5 years) (87,500) (87,500)

Profit after Tax 61,250 70,000

Average Profit after Tax 12,250 14,000

Average Rate of Return (ARR) = Investment Initial

after Tax Profit Average x 100

Machine A = 87,500

250,12x 100 = 14 %

Machine B = 87,500000,14

x100= 16%

Ranking of Machine as per ARR is Machine B : Rank -1

Machine A : Rank -2

Alternate Solution

Based on Average Investment

Average Rate of Return (ARR) = Investment Average

after Tax Profit Averagex 100

Average Investment =

= ̀ 43750 (for both machines)

Initial Investment + salvage value

2

Page 75: EXECUTIVE PROGRAMME (New Syllabus) · GUIDELINE ANSWERS EXECUTIVE PROGRAMME (New Syllabus) JUNE 2019 MODULE 2 ICSI House, 22, Institutional Area, Lodi Road, New Delhi 110 003 Phones:

73 EP-FSM–June 2019

Machine A = 43,75012,250

x 100= 28 %

Machine B = 43,75014,000

x 100= 32%

Ranking of Machine as per ARR is Machine B : Rank -1

Machine A : Rank -2

Alternate Solution

Year Machine A Machine B

CFAT Depreciation EAT CFAT Depreciation EAT

1 26,250 17,500 8,750 8,750 17,500 (8,750)

2 35,000 17,500 17,500 26,250 17,500 8,750

3 43,750 17,500 26,250 35,000 17,500 17,500

4 26,250 17,500 8,750 52,500 17,500 35,000

5 17,500 17,500 - 35,000 17,500 17,500

Average EAT = 61,250/5 = 12,250 Average EAT = 70,000/5 = 14,000

Average Investment = ½ of (Cost of Average Investment = ½ of (Cost ofMachine – Salvage Value) + Machine – Salvage Value) + SalvageSalvage Value Value

= ½ (87,500-0) = 43,750 = ½ (87,500-0) = 43,750

ARR on Initial Investment ARR on Initial Investment

Investment Initialafter Tax Profit Average

x 100Investment Initial

after Tax Profit Average x 100

Machine A = 87,50012,250

x 100 = 14 % = 87,50014,000

x100= 16%

ARR on Average Investment ARR on Average Investment

Investment Averageafter Tax Profit Average

x 100 Investment Averageafter Tax Profit Average

x 100

= 43,75012,250

x 100= 28 % = 43,75014,000

x 100= 32%

Ranking of Machine as per ARR is Machine B : Rank -1 Machine A : Rank -2

Page 76: EXECUTIVE PROGRAMME (New Syllabus) · GUIDELINE ANSWERS EXECUTIVE PROGRAMME (New Syllabus) JUNE 2019 MODULE 2 ICSI House, 22, Institutional Area, Lodi Road, New Delhi 110 003 Phones:

EP–FSM–June 2019 74

Answer 2(e)

Calculation of beta co-efficient

Standard deviation ( s) = 3%

Correlation co-efficient (rm)= 0.70

Market Standard Deviation (m) = 2.8%

Beta Coefficient = (rm x s) ÷ m = 0.70 x 3 ÷ 2.8 = 0.75

Rm= 20 % Rf = 10 %

Expected rate of return = Rf + ß (Rm-Rf)

Rf = Risk free rate of Return

ß = Risk Factor

Rm = Market Return

= 10% +0.75 (20-10)

= 17.5 %

Answer 2A(i)

Cost of debt (Kd) = 0.08 (1 – 0.40) = 0.048 or 4.8%

Cost of equity (Ke) = (Earnings per share/market price per share) x 100

= x 100

5018

x 100 = 12%

Calculation of WACC (based on market value)

Source Amount (`) Weight Specific cost Weighted specificof capital cost of capital

Equity sharecapital 22,50,000 0.602 0.12 0.07224

Debt 10,40,000 0.278 0.048 0.01334

Generalreserves 4,50,000 0.120 0.12 0.0144

37,40,000 1.00 0.09998

Therefore, the weighted average cost of capital is 0.09998 or 10%

Note : Equity share capital and General reserves are divided in the ratio of 5:1 on thebasis of their book value.

[(Rs. 6,23,000 - Rs. 83,200)(1 - 0.40)]/18000Rs. 150

Page 77: EXECUTIVE PROGRAMME (New Syllabus) · GUIDELINE ANSWERS EXECUTIVE PROGRAMME (New Syllabus) JUNE 2019 MODULE 2 ICSI House, 22, Institutional Area, Lodi Road, New Delhi 110 003 Phones:

75 EP-FSM–June 2019

Alternative Solution

Particulars Amount

EBIT 6,23,000

Less : Interest (83,200)

EBT 5,39,800

Less : Tax (2,15,920)

Profit available for equity shareholders 3,23,880

Cost of debt (Kd) = 0.08 (1 – 0.40) = 0.048 or 4.8%

OR 83,200 (1-0.4) ÷ 10,40,000 = 4.8%

Cost of equity (Ke) = (Earnings per share/market price per share) x 100

Earning per share = 3, 23,880 ÷ 18,000 = ̀ 18 per share

= (18 ÷ 150) x 100 = 12 %

Calculation of WACC (based on market value)

Source Amount (`) Weight Specific cost Weighted specificof capital cost of capital

Equity sharecapital 27,00,000 0.7219 0.12 0.086628

(` 150 x 18000shares)

Debt 10,40,000 0.2781 0.048 0.013348

37,40,000 1.0000 0.099976

WACC = 0.099976 x 100 = 9.998 % or 10%

Answer 2A(ii)Cost of Factoring

Annual Credit Sales `50,00,000 x 72 % `36 lakhs

Monthly Credit Sales `36 lakhs/12 `3 lakhs

Factor Cost

Fee (`3,00,000 x 80%) x 2% `4,800

Commission `3,00,000 x 10% `30,000

(A) `34,800

Page 78: EXECUTIVE PROGRAMME (New Syllabus) · GUIDELINE ANSWERS EXECUTIVE PROGRAMME (New Syllabus) JUNE 2019 MODULE 2 ICSI House, 22, Institutional Area, Lodi Road, New Delhi 110 003 Phones:

EP–FSM–June 2019 76

Less : Savings in Cost:

Management Cost (`48,000/12) `4,000

Savings in Bad Debts (` 3,00,000 x 1 %) `3,000

(B) `7,000

Net Cost of Factoring (per month) (A-B) `27,800

Cost of Bank Advance

Interest (`3,00,000 x 80% x 15%) ÷12 `. 3,000

Processing Fee (`3,00,000 x 2%) `6,000

Bad Debts (`3,00,000 x 1%) `3,000

Management Cost `4,000

Net Cost (Per Month) `16,000

Since cost of Bank Finance is less than the cost of factoring, therefore, it is advisableto accept bank offer.

Answer 2A(iii)

(i) Market’s risk return trade-off = (Rm-Rf) ÷ mWhere, Rm = Market Return

Rf = Risk free rate of Return

m = Standard deviation of market

= (0.09-0.02) ÷ 0.04 = 1.75

(ii) Security Beta

ß = (s ÷ m) x rm

where, ß = Beta factor of investment

s = Standard deviation of investment in security =0.08

m= Market Portfolio standard deviation =0.04

rm= Co-efficient of correlation of security with market portfolio =0.75

ß = (0.08 ÷ 0.04) x 0.75 = 1.5

Question 3

Differentiate between the following :

(a) Forfaiting and Export Factoring(b) Regular dividend policy and Stable dividend policy(c) Net Income Approach and Net Operating Income Approach. (5 marks each)

Page 79: EXECUTIVE PROGRAMME (New Syllabus) · GUIDELINE ANSWERS EXECUTIVE PROGRAMME (New Syllabus) JUNE 2019 MODULE 2 ICSI House, 22, Institutional Area, Lodi Road, New Delhi 110 003 Phones:

77 EP-FSM–June 2019

Answer 3(a)

Forfaiting and Export Factoring

Forfaiting is similar to cross border factoring or export factoring to the extent bothhave common features of non recourse and advance payment. But they differ in severalimportant respects:

a. A forfaiter discounts the entire value of the note/bill but the factor financesbetween 75-85% and retains a factor reserve which is paid after maturity.

b. The availing bank which provides an unconditional and irrevocable guarantee isa critical element in the Forfaiting arrangement whereas in a factoring deal,particularly non-recourse type, the export factor bases his credit decision on thecredit standards of the exporter.

c. Forfaiting is a pure financing arrangement while factoring also includes ledgeradministration, collection and so on.

d. Factoring is essentially a short term financing deal. Forfaiting finances notes/bills arising out of deferred credit transaction spread over three to five years.

e. A factor does not guard against exchange rate fluctuations; a forfaiter chargesa premium for such risk.

Answer 3(b)

Regular dividend policy and Stable dividend policy

Regular dividend policy : In Regular dividend policy the investors get dividend atusual rate. Here, the investors are usually persons who want to get regularly incomes.This type of dividend payment can be maintained only if the company has regular earning.

Merits of Regular Dividend Policy:

a. It helps in creating confidence among the shareholders.

b. It stabilizes the market value of shares.

c. It helps in marinating the goodwill of the company.

d. It helps in giving regular income to the shareholders.

Stable dividend policy : Here the payment of certain sum of money is regularlymade to the shareholders. Constant dividend per share, Constant payout ratio, Stablerupee dividend plus extra dividend are the types of Stable dividend policy.

Merits of stable dividend policy:

a. It helps in creating confidence among the shareholders.

b. It stabilizes the market value of shares.

c. It helps in maintaining the goodwill of the company.

d. It helps in giving regular income to the shareholders.

Page 80: EXECUTIVE PROGRAMME (New Syllabus) · GUIDELINE ANSWERS EXECUTIVE PROGRAMME (New Syllabus) JUNE 2019 MODULE 2 ICSI House, 22, Institutional Area, Lodi Road, New Delhi 110 003 Phones:

EP–FSM–June 2019 78

Answer 3(c)

Net income approach and Net operating income approach

Net income approach

According to this approach there is a relationship between capital structure and thevalue of the firm and therefore, the firm can affect its value by increasing or decreasingthe debt proportion in the overall financial mix. The Net Income Approach makes thefollowing assumptions:

1. The total capital requirement of the firm is given and remains constant.

2. Cost of debt (Kd) is less than cost of equity (Ke).

3. Both Kd and Ke remain constant and any change in financial leverage (i.e., useof more and more debt financing) in the capital structure does not affect the riskperception of the investors.

4. There are no taxes.

Net operating income approach is opposite to the Net income approach. Accordingto NOI Approach, the market value of the firm depends upon the net operating profit orEBIT and the overall cost of capital. The financing mix or the capital structure is irrelevantand does not affect the value of the firm. The NOI Approach makes the followingassumptions:

a. The investors see the firm as a whole and thus capitalize the total earnings ofthe firm to find the value of the firm as a whole.

b. The overall cost of capital KO, of the firm is constant and depends upon thebusiness risk which also is assumed to be unchanged.

c. The cost of debt, Kd, is also taken as constant.

d. The use of more and more debt in the capital structure increases the risk of theshareholders and thus results in the increase in the cost of equity capital i.e, Ke.The increase in Ke is such as to completely offset the benefits of employingcheaper debt, and

e. There are no taxes.

Question 4

(a) ABC Ltd. is a firm that specializes in providing software services to its clients.In order to augment its time-share computer services to its clients, the firm isconsidering the purchase of an additional computer.

In this context, it has two alternatives, as mentioned below :

(i) Purchasing the computer in lieu of making a payment of `24,00,000.

(ii) Entering into a lease agreement with a company with the following necessaryterms and conditions :

(a) The computer has to be leased for a period of 3 years for ̀ 6,00,000 as

Page 81: EXECUTIVE PROGRAMME (New Syllabus) · GUIDELINE ANSWERS EXECUTIVE PROGRAMME (New Syllabus) JUNE 2019 MODULE 2 ICSI House, 22, Institutional Area, Lodi Road, New Delhi 110 003 Phones:

79 EP-FSM–June 2019

annual lease rent in addition to 15% of the revenue accruing from grosstime-share service.

(b) An additional payment of ̀ 5,00,000 needs to be done at the end of thethird year.

(c) Payments as lease rent need to be made at the year end.(d) After the completion of the specified period in the contract, the computer

needs to be handed back to the lessor.As per the company estimates, the computer under review, presently willhave a worth of `12 lakhs at the end of the third year with the forecastrevenues as follows :

Year `

1 24,00,0002 25,50,0003 28,00,000

Other relevant details are as follows :(i) The Annual operating costs (excluding the depreciation/lease rent of

the computer) are `10,00,000 as per estimates with an additional`2,00,000 being involved in start-up and training costs at the beginningof the first year.

(ii) The aforesaid costs are to be borne by the lessee ABC Ltd. Further, thelessee will borrow at an interest rate of 15% to finance the acquisitionof the computer. Repayments for the same have to be done as per theschedule specified below :Year-end Principal (`) Interest (`) Total (`)

1 6,00,000 3,60,000 9,60,0002 9,00,000 2,70,000 11,70,0003 9,00,000 1,35,000 10,35,000

In order to depreciate its assets, ABC Ltd. employs the straight line methodand makes a payment of 50% tax on its income.You, as a company secretary, are approached by ABC Ltd. to give adviceregarding the choice of the more beneficial alternative, with proper andjustified reasons.Consider Present Value Factor at 9%Year Present value factor at 9% Present value factor at 15%1 0.917 0.8702 0.842 0.7563 0.772 0.658

(8 marks)(b) Hindustan Chemical Ltd. has operating and capital structure with financial plan

A and B under situations I and II. Annual production and sales are 75% ofinstalled capacity.

Page 82: EXECUTIVE PROGRAMME (New Syllabus) · GUIDELINE ANSWERS EXECUTIVE PROGRAMME (New Syllabus) JUNE 2019 MODULE 2 ICSI House, 22, Institutional Area, Lodi Road, New Delhi 110 003 Phones:

EP–FSM–June 2019 80

Compute the operating, financial and combined leverage of financial plan A andB under situation I and II. And also interpret combined leverage with referenceto highest value and lowest value under both financial plans as well as situationsfrom the following information :

Installed capacity 10,000 unitsSelling price `40Variable cost per unit `20Fixed costsSituation I `15,000Situation II `18,000Capital Structure

Financial Plan A B ` `

Equity 30,000 40,000Debt (12%) 40,000 30,000Total 70,000 70,000 (7 marks)

Answer 4 (a)

(Amount in `)

Present Value of cash out flows under lease alternative

Year Payment under lease contract Total Tax Shield Net Cash PV factor Total PVLease 15% gross Lump Payment @ 50 % on Outflows at 9 %Rent Revenue sum lease

pay-ment

1 6,00,000 3,60,000 - 9,60,000 4,80,000 4,80,000 0.917 4,40,160

2 6,00,000 3,82,500 - 9,82,500 4,91,250 4,91,250 0.842 4,13,632.50

3 6,00,000 4,20,000 5,00,000 15,20,000 7,60,000 7,60,000 0.772 5,86,720

14,40,512.50

Present Value of cash out flows under Buying/Borrowing alternative

Year Installment Payment Tax advantage on Net Cash PV factor Total Principal Interest Total Interest Depreciation Outflows at 9 % PV

1 6,00,000 3,60,000 9,60,000 1,80,000 2,00,000 5,80,000 0.917 5,31,860

2 9,00,000 2,70,000 11,70,000 1,35,000 2,00,000 8,35,000 0.842 7,03,070

3 9,00,000 1,35,000 10,35,000 67,500 2,00,000 7,67,500 0.772 5,92,510

18,27,440

Salvage Value 12,00,000 0.772 (9,26,400)

9,01,040

Page 83: EXECUTIVE PROGRAMME (New Syllabus) · GUIDELINE ANSWERS EXECUTIVE PROGRAMME (New Syllabus) JUNE 2019 MODULE 2 ICSI House, 22, Institutional Area, Lodi Road, New Delhi 110 003 Phones:

81 EP-FSM–June 2019

Recommendation : PV of cash outflows under leasing is ̀ 14,40,512.50 which is morethan the PV of cash outflows under buying/borrowing of ̀ 9,01,040. Therefore, the companyis advised to opt buying/borrowing.

Depreciation for 3 years = (`24,00,000 - `12,00,000) = `12,00,000; Depreciation peryear = ̀ 4,00,000.

Tax savings on depreciation= 50% of `4,00,000 = ̀ 2,00,000

Note: Since the annual operating costs and training costs are same under both thealternatives. Hence, this cost is irrelevant in the decision of purchase or lease. So it isnot included in the calculation of cash outflows.

Answer 4 (b)

Annual production and sales 75% of 10,000 units installed capacity = 7,500 units

Contribution per unit= Selling price per unit –Variable cost per unit i.e `40 - `20 = `20

Total Contribution is 7,500 x 20 = ̀ 1,50,000(Amount is `)

Financial Plan A Financial Plan B

Situation I Situation II Situation I Situation II

Contribution 1,50,000 1,50,000 1,50,000 1,50,000

Less : Fixed Costs 15,000 18,000 15,000 18,000

Operating profit or EBIT 1,35,000 1,32,000 1,35,000 1,32,000

12% Interest on Debt

On ̀ 40,000 4,800 4,800 - -

On ̀ 30,000 - - 3,600 3,600

Earnings before tax 1,30,200 1,27,200 1,31,400 1,28,400

Operating Leverage (OL)(Contribution/EBIT) 1.11 1.14 1.11 1.14

Financial Leverage (FL)

EBIT/EBT 1.04 1.04 1.03 1.03

Combined Leverage (OL x FL) 1.15 1.19 1.14 1.17

Highest and least value of combined leverage

Highest Value - 1.19 under situation II of Financial Plan A

Situation II of Financial Plan A at 1.19 showing highest risk (both operating risk aswell as financial risks are high). As a result any change in sales will have higher impacton EPS.

Page 84: EXECUTIVE PROGRAMME (New Syllabus) · GUIDELINE ANSWERS EXECUTIVE PROGRAMME (New Syllabus) JUNE 2019 MODULE 2 ICSI House, 22, Institutional Area, Lodi Road, New Delhi 110 003 Phones:

EP–FSM–June 2019 82

Least Value – 1.14 under situation I of Financial Plan B

Situation I of Financial Plan B at 1.14 showing lowest combined risk (both operatingrisk as well as financial risks are low). As a result any change in sales will have lesserimpact on EPS.

PART II

Question 5

Case Study :

About the company

Black Diamond Ltd. (BDL) is a public limited company in Country-B engaged inmining and production of coal and coal products. The mission of the company is “tobe a partner in the economy by producing and fulfilling the energy demand of thecountry by providing quality coal in economical and sustainable manner”. Thecompany is listed on a stock exchange of the Country B. Shares of the companyare held by the Govt. of Country-B, Mutual funds & FII and general public in proportionof 75%, 12% and 13% respectively. During the financial year 2018-19, the companyhas produced and dispatched 500 million tonnes of coal. Of this 76% to powerplants, 2% to steel plant and rest to other sectors.

Coal is the prime source of energy and fulfills approx. 70% of energy demand of thecountry B. The thermal power plants, steel manufacturing plants, cement and fertilizerplants are the major consumers of coal.

The country-B has a large reserve of coal but the quality and cost are major concern.In the last few years there has been increase in import of coal by steel and powerplants. The steel plants import coal due to quality and cost advantage over domesticcoal. As per a study report conducted by the BDL, good quality of coal can beproduced only through deep and underground mining. Underground mining requireshighly automated technology.

Challenges before BDL

For adopting deep underground mining the BDL is required to acquire land for newmines and latest automated machines for mining. Land acquisition in country-B hasbeen a hard task due to land acquisition related laws and protests by the localcommunity and NGOs as BDL has acres of idle land in the form of used abandonedmines which cannot be used for agricultural activities or human rehabilitation due tothe risk of landslides. BDL is a labour intensive company with low productivity.Employee cost is the major cost of BDL which accounts for 55% of total cost. BDLis second largest employer in country B.

Till recent past, the BDL was the only company which was allowed for mining andmarketing of the coal in the country but with the changing economic scenario and tomeet the growing energy demand the Govt. of Country-B is mulling to allot coalblocks to steel plant and thermal power plant for captive consumption.

The Govt. of the Country B is committed to boost manufacturing and industrialinfrastructure of the country in next 10 years. Power and steel sector would be

Page 85: EXECUTIVE PROGRAMME (New Syllabus) · GUIDELINE ANSWERS EXECUTIVE PROGRAMME (New Syllabus) JUNE 2019 MODULE 2 ICSI House, 22, Institutional Area, Lodi Road, New Delhi 110 003 Phones:

83 EP-FSM–June 2019

witnessing a multifold increase in demand not only from Govt. schemes but alsofrom private and domestic consumers.

To meet the energy demand, the Govt. has reduced the duty on imported coal. TheGovt. being a signatory of Paris agreement on climate change, is committed forsustainable development by reducing carbon emission caused by usage of fossilfuels. It has entered into various technological MoUs to install clean energy sourceslike nuclear, solar and wind in near future.

Required :

(i) Identify and describe the five forces of Porter with respect to BDL.

(ii) Identify the stakeholders in BDL. Discuss the strategy which BDL should adoptto prioritise the stakeholders.

(iii) Explain the strategy BDL should adopt to survive and gain competitiveadvantage.

(iv) Prepare a SWOT analysis for BDL.

(v) If BDL wants to change itself from a labour intensive organization to a moderntechnology driven organization, state how BDL can manage the resistance tochange from employees related unions ? (4 marks each)

Answer 5(i)

The five forces model which was created by Michael Porter is an analytical tool thatuses industry forces that uses five industry forces to determine the intensity of competitionin an industry and its profitability level. The five forces in present case of BDL is describedas under:

a. Industry Rivalry/Competition (Low)Industry rivalry for the BDL is low as it is the only company which is allowed formining and marketing of the coal in the country and most of the energyrequirement of the country is fulfilled by coal. However, increase in import alongwith reduced import duty may be a potential threat.

b. Threat of New Entrants (Medium/high)With the changing economic scenario and to meet the growing energy demandthe Govt. of Country B is mulling to allot coal blocks to steel plant and thermalpower plant for captive consumption which may be a cause of worry for BDL innear future.

c. Threat of Substitutes (Low)At present, coal is the prime source of energy and fulfills approx. 70 % ofenergy demand of the country B. There are various substitutes of coal likeNatural gas, solar, wind, nuclear energy etc. Since the Government of countryB is a signatory of Paris agreement on climate change and entered technologicalMoUs to install clean energy sources like nuclear, solar and wind energy, thethreat of substitutes may increase in future.

d. Bargaining power of buyers (Low/Medium)The thermal power plants, steel manufacturing plants, cement and fertilizer

Page 86: EXECUTIVE PROGRAMME (New Syllabus) · GUIDELINE ANSWERS EXECUTIVE PROGRAMME (New Syllabus) JUNE 2019 MODULE 2 ICSI House, 22, Institutional Area, Lodi Road, New Delhi 110 003 Phones:

EP–FSM–June 2019 84

plants are the major customers of BDL. At present, there is no other alternativeavailable with the customer as the BDL is the only coal producing and marketingcompany. But in near future bargaining power of buyer may increase as theGovt. of country B is mulling to allot coal blocks to steel and thermal powerplant.

e. Bargaining power of suppliers (Medium/High)Being labour intensive company, suppliers of consumables, work contractorsand the employees can be classified as suppliers for BDL. When the technologywill change, power of supplier who are supplying for specific parts for heavymachineries, explosive and power etc. may increase.

Answer 5(ii)

Stakeholders in BDL are as under:

• Investors (Government, Mutual Funds and FII and General Public)

• Customers i.e thermal power plants, steel manufacturing plants, cement andfertilizer plants

• Suppliers

• Employees of BDL, Union and Associations

• Regulators as Administrative body (Ministry of Mines, Ministry of Environment,Forest and Climate Change, Ministry of Labour and Employment)

Priortising of Stakeholders

• BDL should give priority to Regulators who have financial, regulatory and positionalpowers to allot coal blocks to other companies and can influence the businessof company.

• Being the labour intensive company, employees of BDL should be second priorityand be satisfied as the functioning of the company is totally dependent onthem.

• Customers should also be then priortised because in changing economic scenarioand to meet the growing energy demand, the Government may allot coal blocksto others for captive consumption.

• Further, suppliers should also not be ignored as the functioning of company isdependent on them by providing the supplies. Last but not the least, thestakeholders who are neither having excessive interest nor power to influenceshould be monitored for their action and handles appropriately.

Answer 5(iii)

BDL should adopt the Porter’s Generic Strategies i.e Cost leadership, Productdifferentiation and Focus on Niche, which are described below:

Cost Leadership Strategy : BDL being the leader in its market should adopt thecost leadership strategy. Power plants are main customers of BDL. Keeping this in viewthe quality of coal, power plants are importing coal from last few years. It is necessary

Page 87: EXECUTIVE PROGRAMME (New Syllabus) · GUIDELINE ANSWERS EXECUTIVE PROGRAMME (New Syllabus) JUNE 2019 MODULE 2 ICSI House, 22, Institutional Area, Lodi Road, New Delhi 110 003 Phones:

85 EP-FSM–June 2019

to maintain the power plant customers by providing the good quality coal by using highlyautomated technology. Initially, it may require high capital investment but it will offer thevarious benefits like decreasing the total production cost, maintaining of customers,less pollution, low manpower cost etc.

Product Differentiation Strategy : BDL can produce the good quality coal throughdeep and underground mining using highly automated technology. In view to meet thegovernment commitment for sustainable development by reducing carbon emissioncaused by usage of fossil fuels, the company may enter in to solar and wind energyproduction on idle abandoned land.

Niche/ Focus Strategy : Company may also use focus strategy to meet the uniquerequirements of company like quality coal which consist of 76 % of total customer base,which is only possible by installing highly automated machine for creating competitiveadvantage.

Answer 5(iv)

SWOT Analysis for BDL

Strengths Weaknesses Opportunities Threats

• Monopoly in • Labour intensive • Country has large • Increasing demand ofdomestic market company reserve of coal quality control by steel

• Strong coal • Bad Quality of coal • Extracting good and power plants whichdemand for power plant which quality coal through is presently not available

• Government is 76% of customer deep mining with BDL.backed organi- base • Abandoned land for • Government plan to allotsation with 75% • High cost of diversification i.e coal blocks to steel andshares production solar and wind thermal power plant may

• Rich experience • Lack of highly energy reduce consumption ofin mining automated • Planning of govern- coal provided by BDL.

• Being listed entity, technology ment to boost • The governmentit can raise more manufacturing and commitment to promotefunds from market industrial infra- non-conventional energyfor technology structure will hamper the coalupgradation demand in future.

• Employee cost with lowproductivity can be a bigthreat due to lack ofcompetitive advantage.

Answer 5(v)

BDL can manage the resistance to change from employees related unions by followingways:

i. Education and Communication : If misinformation and lack of information createbarriers to managing change, education and communication might be appropriate.It requires an atmosphere of mutual trust and confidence and respect betweenmanagers and employees.

ii. Participation : Participation helps to give people in organizational change afeeling of importance. It creates the feelings among the employees that thedecision is their own. They realise that the change process is a must.

Page 88: EXECUTIVE PROGRAMME (New Syllabus) · GUIDELINE ANSWERS EXECUTIVE PROGRAMME (New Syllabus) JUNE 2019 MODULE 2 ICSI House, 22, Institutional Area, Lodi Road, New Delhi 110 003 Phones:

EP–FSM–June 2019 86

iii. Obtaining commitment : Commitment to take part in changed programme canbe obtained on one to one basis from employees. However, getting a person tocommit himself in private to a changed programme may yield fewer results thanif he voluntarily and publicly gives his commitment to an idea of change.

iv. Leadership : A transformational leader can use personal reasons for changewithout arousing resistance. An effective leader tries to change the psychologicalneeds of his followers.

v. Training and Psychological Counselling : Management can change the basicvalues of the people by training and psychological counselling. People shouldbe educated to become familiar with change, its process, and working.

vi. Coercion or Edict : Coercion or edict is the imposition of change or the issuingof directives about change. It is the explicit use of power. Coercion is the leastsuccessful style of managing change except in a state of crisis or confusion.

Attempt all parts of either Q. No. 6 or Q. No. 6A

Question 6

(a) ‘Planning is the blue print of future course of action’. Discuss with the significanceof planning.

(b) ‘A company secretary in today’s era while discharging his/her professionalobligation has to perform several key roles which are also integral componentsof strategic management’. Discuss briefly about compliance and representationroles.

(c) ‘Logistics strategy plan is an important part of formulation of strategy’. What arethe major elements of Strategy Plan ?

(d) ‘Despite various advantages, divisional structure form of organisation has certaindisadvantages’. Discuss, highlighting major disadvantages. (5 marks each)

OR (Alternate question to Q. No. 6)

Question 6A

Comment on the following :

(i) Logistics is not confined to tactical decision about transportation andwarehousing.

(ii) An effective implementation of strategy in an organisation needs multiplesupporting factors.

(iii) Porter has pronounced three generic strategies, namely “Cost Leadership”,“Differentiation”, and “Focus”.

(iv) Behavioural implementation is concerned with those aspects of strategyimplementation which have influence on the behaviour of the people in theorganisation.

(v) Failure of a firm is technical if it is unable to meet its current obligations.(4 marks each)

Page 89: EXECUTIVE PROGRAMME (New Syllabus) · GUIDELINE ANSWERS EXECUTIVE PROGRAMME (New Syllabus) JUNE 2019 MODULE 2 ICSI House, 22, Institutional Area, Lodi Road, New Delhi 110 003 Phones:

87 EP-FSM–June 2019

Answer 6(a)

Planning is significant because the planning provides the organization a better senseof what it wants to achieve and how to achieve it. In effect, planning ensures the properutilization of the available resources and the capability to comprehend how these shouldbe used in order to achieve the goal. A key part of planning is also the vital role it playsin pacifying risks. When management plans for the tasks ahead, they are looking at thesituation and detailing the probable difficulties ahead.

Answer 6(b)

A company secretary in today’s era while discharging his or her professionalobligations has to perform several key roles which are also integral components ofstrategic management. A brief on compliance and representation roles is as follows:

Compliances role : In current scenario a business has to adhere to various laws andregulations failing which may invite various legal hassles. A company secretary is requiredto ensure compliance with various laws and regulations and for doing so he / she shouldbe conversant with the laws as well as the amendments that take place. For instance, inIndian context a company secretary has to ensure compliances of the following laws butnot limited to- Companies Act; SEBI Act, Securities Contracts (Regulation) Act andrules and regulations made there under; Foreign Exchange Management Act; ConsumerProtection Act; Depositories Act; Environment and Pollution Control Laws; Labour andIndustrial Laws etc.

Representation role : A Company Secretary has to represent before various tribunalsand courts in order to present the legal issue of the company. In India, a CompanySecretary appears before the following legal bodies- National Company Law Tribunal(NCLT); National Company Law Appellate Tribunal (NCLAT); Competition Commissionof India (CCI); Registrar of Companies; Tax Tribunals etc.

Answer 6(c)

Major elements of Logistics Strategy plan are as under:

1. Customer service policy – the appropriate level of service for customers, byproduct group or market segment; considering order fulfillment requirements,enquiry and investigation capability and the available information. The customerservice policy informs the nodes and links of the supply network

2. Inventory location policy (Supply Network nodes) – Centralized or decentralizedinventory; whether to differentiate facilities by fast and slow moving stock; locationof sites; use of specific technologies and layouts; company-owned or contractedfacilities etc.

3. Inventory policy – Form and function of inventory by location; the appropriateamount of stock to hold for various groups of inventory; planning structure thatlinks outbound and inbound materials

4. Cost plan – Trade-off analysis between cost and service level requirements;cost of Logistics operations

5. Transport and distribution (Supply Network links) policy – affected by whether

Page 90: EXECUTIVE PROGRAMME (New Syllabus) · GUIDELINE ANSWERS EXECUTIVE PROGRAMME (New Syllabus) JUNE 2019 MODULE 2 ICSI House, 22, Institutional Area, Lodi Road, New Delhi 110 003 Phones:

EP–FSM–June 2019 88

enterprise imports or exports and the size and structure of conurbations beingserved. This incorporates transport modes, delivery pattern and storage locationconsiderations, based on the time taken for deliveries.

6. IT and Communications capability : Technologies (including software) that willbe internally developed; buy planning and scheduling applications from singlesupplier or obtain ‘best of breed’ applications.

7. Logistics organisation structure : Function or flow based; allocation ofresponsibilities and managed or self-managed teams

8. Logistics Targets and metrics : Measures of performance and achievementtargets; operations improvements process and management.

Answer 6(d)

Despite various advantages, divisional structure form of organization has certaindisadvantages also. Some of them are as under:

i. Costly affair : It incurs extra costs since each division has its own set of functionaldepartments and attached staff.

ii. Competition between divisions : Divisionalisation and compartmentalization leadseach division to compete for more resources leading to competition among thedivisions due to which one department suffers at the cost of another.

iii. Coordination problem : It is difficult to manage as a result of more divisions anddepartments

iv. Change of priorities : The service departments such as research and developmentin order to get higher and quick returns on investments.

Answer 6A(i)

Logistics is not confined to tactical decisions about transportation and warehousing.Longer-term decisions are needed to put in place the capabilities that ensure that logisticsplays a full role in supporting a company’s products in the market place. When a companycreates a logistics strategy it is defining the service levels at which its logisticsorganization is at its most cost effective. Because supply chains are constantly changingand evolving, a company may develop a number of logistics strategies for specificproduct lines, specific countries, or specific customers. Apart from transportation andwarehousing, logistics strategy deals with the following:

– Sources of Raw material and Components available

– Number of Manufacturing locations, and the products manufactured therein

– Nature of material handling equipments used

– Nature of distribution facilities

– Method for deploying inventory in the logistics network

Answer 6A(ii)

The development and selection of strategies to pursue in an organization is consideredeasier and less time consuming than implementing these strategies once they have

Page 91: EXECUTIVE PROGRAMME (New Syllabus) · GUIDELINE ANSWERS EXECUTIVE PROGRAMME (New Syllabus) JUNE 2019 MODULE 2 ICSI House, 22, Institutional Area, Lodi Road, New Delhi 110 003 Phones:

89 EP-FSM–June 2019

been chosen. An effective implementation of strategy in an organization needs multiplesupporting factors, which are as under:

i. Action Planning - Organizations to be successful in strategy implementationneed to develop a detailed action plan i.e., chronological lists of action steps(tactics) which add the necessary details to strategies and assign responsibilityto specific individual or group for accomplishing those actions.

ii. Organizational Structure - Successful strategists should also give proper thoughtto their organizational structure and see whether the current structure isappropriate for their intended strategy because different structures suit theimplementation of different strategies

iii. Human Resource Factors - Human Resource factors through framing strategicplan play a vital role in successful implementation of strategies in an organization.The consideration of human resources requires the management to think aboutthe organization’s communication needs. Further, managers successful atimplementation of different strategies are aware of the effects of each newstrategy that will have on their human resource needs.

iv. Annual Business Plan - Organizations successful at implementation are wellaware of their need to fund their intended strategies.

v. Monitoring and Control - Monitoring and controlling the plan covers a list ofoptions to get back on course if company should veer off.

Answer 6A(iii)

Michael Porter pronounced the three generic strategies namely “Cost Leadership”(no frills), “Differentiation” (creating uniquely desirable products and services) and “Focus”(offering a specialized service in a niche market).

The Cost Leadership Strategy - This strategy also involves the firm winning marketshare by appealing to cost-conscious or price-sensitive customers. This is achieved byhaving the lowest prices in the target market segment, or at least the lowest price tovalue ratio (price compared to what customers receive).

The Differentiation Strategy - A differentiation strategy is appropriate where thetarget customer segment is not price-sensitive, the market is competitive or saturated,customers have very specific needs which are possibly under-served, and the firm hasunique resources and capabilities which enable it to satisfy these needs in ways that aredifficult to initiate.

The Focus Strategy - The focus strategy is also known as ‘niche’ strategy. This isso because, companies adopt focus strategies, focus on niche markets and, by get holdof the dynamics of such niche market and unique requirements of its customers. Basedon such understanding, they develop exclusively low-cost products particularly for suchniche market. Due to this, a strong brand loyalty is developed with its customers makingit difficult for competitors to enter.

Answer 6A(iv)

Since the organization is basically a deliberate creation of human beings for certain

Page 92: EXECUTIVE PROGRAMME (New Syllabus) · GUIDELINE ANSWERS EXECUTIVE PROGRAMME (New Syllabus) JUNE 2019 MODULE 2 ICSI House, 22, Institutional Area, Lodi Road, New Delhi 110 003 Phones:

EP–FSM–June 2019 90

specified objectives, the activities and behavior of its members need to be directed incertain way. Some of the issues which are relevant for behavioural implementation ofstrategy in an organization are as under:

Leadership - Leaders are the vital aspect of an organization which helps to copewith the changes by ensuring that plans and policies are formulated and implemented asdesired. Leadership is basically the ability to persuade others to achieve the definedobjectives willingly and enthusiastically. A strategic leadership involves the process oftransforming an organization with the help of its people so as to put it in a uniqueposition.

Organizational Culture - Organizational culture affects strategy implementation as itprovides a framework within which the behavior of the members takes place. It is definedas a set of assumptions, beliefs, values and norms which are shared by people andgroups in the organization and control the way they interact with each other and withstakeholder outside the organization.

Values and Ethics - Values are convictions and a framework of philosophy of anindividual on the basis of which one would judge what is good or bad. Business ethicsgenerates from: (a) value forming institutions; (b) organizational values and goals; (c)colleagues; and (d) professional code of conduct.

Answer 6A(v)

Failure of a firm is technical if it is unable to meet its current obligations. The failurecould be temporary and might be remediable. When liabilities exceed assets i.e. the networth becomes negative, resulting into bankruptcy. Technical bankruptcy can beascertained by comparing current assets and current liabilities i.e. current ratio or quickratio. A negative working capital indicates that short term borrowings be used for financinglong term assets which may lead to insolvency in future. For example, a high level ofinterest payment due to high proportion of debt in the capital structure it may result intoinadequate cash flows. If the situation persists for a long time may result in financialcrises due to pressure from creditors and creditors would take their loan back from thecompany. It will impact the share price of the company also as investors will withdrawtheir funds. In short, company may find itself in the situation of financial distress.

***